Sunteți pe pagina 1din 93

LSAT TEST:

Analytical Reasoning:
Directions :All LSAT sample analytical resoning questions are based on a passage or set of conditions. While answering a few of the questions, you would find it useful to draw a rough diagram. To answer any LSAT sample analytical reasoning question choose the answer you think is most appropriate among the given options. Questions 1- 3 Three men (Tom, Peter and Jack) and three women (Eliza, Anne and Karen) are spending a few months at a hillside. They are to stay in a row of nine cottages, each one living in his or her own cottage. There are no others staying in the same row of houses. 1. Anne, Tom and Jack do not want to stay in any cottage, which is at the end of the row. 2. Eliza and Anne are unwilling to stay besides any occupied cottage.. 3. Karen is next to Peter and Jack. 4. Between Anne and Jack's cottage there is just one vacant house. 5. None of the girls occupy adjacent cottages. 6. The house occupied by Tom is next to an end cottage. 1. Which of the above statements can be said to have been derived from two other statements ? A. Statement 1 B. Statement 2 C. Statement 3 D. Statement 5 E. Statement 6 Ans : D 2. How many of them occupy cottages next to a vacant cottage ? A. 2 B. 3 C. 4 D. 5 E. 6 Ans : C

3. Which among these statement(s) are true ? I. Anne is between Eliza and Jack. II. At the most four persons can have occupied cottages on either side of them. . III. Tom stays besides Peter. D. I only E. II only F. I and III only G. II and III only H. I, II and III Ans : C Questions 4 - 7 An employee has been assigned the task of allotting offices to six of the staff members. The offices are numbered 1 - 6. The offices are arranged in a row and they are separated from each other by six foot high dividers. Hence voices, sounds and cigarette smoke flow easily from one office to another. Miss Robert's needs to use the telephone quite often throughout the day. Mr. Mike and Mr. Brown need adjacent offices as they need to consult each other often while working. Miss. Hardy, is a senior employee and has to be allotted the office number 5, having the biggest window. . Mr. Donald requires silence in the offices next to his. Mr. Tim, Mr. Mike and Mr. Donald are all smokers. Miss Hardy finds tobacco smoke allergic and consecutively the offices next to hers to be occupied by non-smokers. Unless specifically stated all the employees maintain an atmosphere of silence during office hours. 4. The ideal candidate to occupy the office furthest from Mr. Brown would be A. Miss Hardy B. Mr. Mike C. Mr. Tim D. Mr. Donald E. Mr. Robert Ans : D 5. The three employees who are smokers should be seated in the offices. A. 1, 2 and 4 B. 2, 3 and 6 C. 1, 2 and 3 D. 1, 2 and 3 E. 1, 2 and 6 Ans : D

6. The ideal office for Mr. Mike would be. A. 2 B. 6 C. 1 D. 3 E. 4 Ans : D 7. In the event of what occurrence, within a period of one month since the assignment of the offices, would a request for a change in office be put forth by one or more employees ? A. Mr. Donald quitting smoking. B. The installation of a noisy teletype machine by Miss Hardy in her office. C. Mr. Robert's needing silence in the office (s) next to her own. . D. Mr. Brown suffering from laryngitis. E. Mr. Tim taking over the duties formerly taken care of by Miss. Robert. . Ans : E In an experiment conducted at a laboratory, 160 white mice were injected with Serum D. 160 other white mice were injected with a harmless sugar solution .In two weeks time 39% of the white mice, who were injected with Serum D contracted the highly contagious and often fatal disease, jungle fever. Hence, it can be concluded that jungle fever is caused by some elements similar to the elements in Serum D. 8. The above discussion would be weakened most severely in case it is shown that A. People contracting jungle fever are usually the victims of the bite of the South American Lesser Hooded Viper. B. One among the 160 white mice had already contracted jungle fever prior to the laboratory experiment. C. The natural habitats of white mice does not contain any of the elements found in Serum D. D. The scientists administered the injections being ignorant of the contents of the solutions used. E. The 160 white mice used in the laboratory experiment were kept isolated from each other. Ans : B 9. The above argument would be highly empowered in case it were shown that: A. Some of the elements in Serum D are extracted from the root of a certain poisonous jungle wildflower. B. Within a period of two weeks about 40% of the white mice, who were injected with a harmless sugar solution also contracted jungle fever.

C. Almost all the white mice died within a period of two days after the first symptoms appeared. D. Normally the rate of jungle fever among white mice is less than 0.01%. E. Invariably the blood of the victims of jungle fever victims contains a high level of a certain toxic substance also found in serum D. Ans : E 10. Distribution of leaflets and delivering speeches on government property should be outlawed. Radicals and fanatics have no right to use public property when peddling their unsavory views. The argument above is based on the postulate A. The general public has a special concern in the free exchange of different political views. B. Radicals and fanatics prefer the use of public property while propagating their viewpoint. C. Every person who hands out leaflets and delivers speeches is a radical or fanatic. D. Legal constraints which are applicable to one group need not be equally applicable to all. E. Any political activity, which hinders the proper functioning of the government should not be protected by the law. Ans : C Questions 11 - 12 Successfully adjusting to one's environment leads to happiness. War at a universal level war destroys the weaker people, who are the most unable to adjust to their environment. Thus, war at the universal level puts weaklings out of their misery and allows more space for their predators to enjoy life in a better manner. As those actions have to be performed, which maximize the level of happiness of the greatest number, war at a universal level should take place. 11. What response would the author of the above discussion come up with, in the case of the objection that the weaklings far exceed strong people? I. He would respond with the statement that the person making the objection is a weakling. II. He would respond by saying that weaklings will be miserable no matter what happens. III. He would respond with the statement that the strong would be frustrated if the weaklings are destroyed. D. I only E. II only F. III only G. I and II only H. II and III only

Ans : E 12. The author's discussion would be greatly if he agreed to which of the following? . Technology could change the environment. I. War at the universal level would be an integral part of the environment. II. It is possible for the strong to survive without suppressing the weak. C. I only D. II only E. III only F. I and III only G. I, II and III only Ans : A Come back with us to the real America leaving behind the turmoil of civilization. The real America is still inhabited by the eagle, the buffalo, the mountain lion and elk; it is still spacious, sprawling and majestic. Experience the freedom and serenity still to be found in 13. Choose the best option to complete the above statement: A. the natural beauty of our land B. the fascinating urban centers C. the wild terrain of Africa D. one's own subconscious E. the great sprawling cities of the Southwest Ans : A 14. The above paragraph is most likely to appear in which of the following? A. A Hunter's Guide to The United States B. Exploring the Great Outdoors C. The Quiet Beauty of Alaska D. How the Eagle Became Extinct E. Returning to America Ans : D 15. When I am elected, I will work towards effecting those changes for which I have been fighting all these years. We will work together to do away with the bureaucratic bogs which have existed ever since my opponent took office. Everyone of you knows what I stand for; I invite my opponent to ... For completion of the above statement choose the best option: A. hand in his resignation graciously B. make his stance clear C. stop lying to the public

D. get our city more federal aid E. extend his support to me Ans : B 16. We can never make our beliefs regarding the world certain. Even scientific theory of a most rigorous and well-confirmed nature is likely to change over a decade or even tomorrow. If we refuse to even try to understand, then it is like resigning from the human race. Undoubtedly life of an unexamined kind is worth living in other respects--as it is no mean thing to be a vegetable or an animal. It is also true that a man wishes to see this speculative domain beyond his next dinner. From the above passage it is clear that the author believes that A. B. C. D. E. men would not do well to speculate progress in the scientific field is impossible one should live life with the dictum 'what will be will be' men should ignore their animal needs men are different from animals as far as their reasoning abilities are concerned.

Ans : E Questions 17-21 Two or more essences out of a stock of five essences-- L, M, N, O, and P are used in making all perfumes by a manufacturer. He has learned that for a blend of essences to be agreeable it should comply with all the rules listed below. A perfume containing L, should also contain the essence N, and the quantity of N should be twice as that of L. A perfume containing M, must also have O as one of its components and they should be in equal proportion. A single perfume should never contain N as well as O. O and P should not be used together. A perfume containing the essence P should contain P in such a proportion that the total amount of P present should be greater than the total amount of the other essence or essences used. 17. Among the following which is an agreeable formula for a perfume? A. One part L, one part P B. Two parts M, two parts L C. Three parts N, three parts L D. Four parts O, four parts M E. Five parts P, five parts M Ans : D

18. Adding more amount of essence N will make which of the following perfumes agreeable? A. One part L, one part N, five parts P B. Two parts M, two parts N, two parts P C. One part M, one part N, one part P D. Two parts M, one part N, four parts P E. Two parts N, one part O, three parts P Ans : A 19. Among the following, the addition of which combination would make an unagreeable perfume containing two parts N and one part P agreeable? (A) One part L (B) One part M (C) Two parts N (D) One part O (E) Two parts P Ans : E 20. Among the following which combination cannot be used together in an agreeable perfume containing two or more essences? A. L and M B. L and N C. L and P D. M and O E. P and N Ans : A 21. Among the below mentioned formulas, which can be made agreeable by the eliminating some or all of one essence ? A. One part L, one part M, one part N, four parts P B. One part L, two parts N, one part O, four parts P C. One part L, one part M, one part O, one part P D. Two parts L, two parts N, one part O, two parts P E. Two parts M, one part N, two parts O, three parts P Ans : B

22. Everything that a person does, which is dictated by reason of ignorance is not voluntary. Involuntary actions are those which produce pain and repentance. Incase a man has done something in his ignorance and he does not feel vexed due to his action, he has not acted voluntarily as he was not aware of what he was doing, nor yet involuntarily since he is not pained. After reading this passage we can arrive at the conclusion that:

A. A person is not a voluntary agent, if he acts by reason of ignorance and repents. . B. If an action is done by reason of ignorance and is not voluntary , then it was repented. C. A man is an involuntary agent, if he acts by reason of ignorance. D. Some actions are either involuntary or not voluntary. E. If a man is not a voluntary agent, then he acted by reason of ignorance and repents. Ans : A 23. Everything that God knows necessarily is, because even what we ourselves know necessarily is; and, of course, our knowledge is not as certain as God's knowledge is. But no future contingent thing necessarily is.. Among the following statements, which naturally follows from the above: 1. There are no future contingent things. 2. It is not true that God has knowledge of only necessary things. 3. God has knowledge of no contingent future things. 4. It is not possible for us to know God. 5. God has knowledge of everything. . Ans : C Questions 24 - 25 Some lawyers are of the view that the observation of the intrinsic qualities of pornography in any composition depends on literary criticism and hence it is a matter of opinion. It is rather odd, though, that in a legal connection, serious critics themselves quite often behave as if they believed criticism to be a matter of opinion. Why be a critic - and teach in universities - in case criticism involves nothing but uttering capricious and arbitrary opinions ? 24. In the above argument the author is trying to establish that A. whether a composition can be called pornographic or not is a matter of opinion. . B. it is not a matter of opinion whether a work is pornographic. C. observance of the qualities of pornography is not dependent on literary criticism. D. critics seem hypocritical. E. critics should not teach at universities. Ans : D 25. The above discussion would be weakened if it is pointed out that: A. literary critics are of the opinion that nothing is pornographic. B. lawyers believe that the observance of the qualities of pornography is a matter of opinion, as literary critics are not in agreement in this regard.

C. literary critics are not legal authorities. D. literary critics should not concern themselves with deciding what is pornographic. E. literary critics in the teaching profession at the university level are init only for the money. Ans : B Nine individuals - Z, Y, X, W, V, U, T, S and R - are the only candidates, who can serve on three committees-- A, B and C, and each candidate should serve on exactly one of the committees. Committee A should consist of exactly one member more than committee B. It is possible that there are no members of committee C. Among Z, Y and X none can serve on committee A. Among W, V and U none can serve on committee G. Among T, S and R none can serve on committee C. 26. In case T and Z are the individuals serving on committee B, how many of the nine individuals should serve on committee C? A. 3 B. 4 C. 5 D. 6 E. 7 Ans : B 27. Of the nine individuals, the largest number that can serve together on committee C is A. 9 B. 8 C. 7 D. 6 E. 5 Ans : D 28. In case R is the only individual serving on committee B, which among the following should serve on committee A? A. W and S B. V and U C. V and T D. U and S E. T and S Ans : E

29. In case any of the nine individuals serves on committee C, which among the following should be the candidate to serve on committee A? A. Z B. Y C. W D. T E. S Ans : C 30. In case T, S and X are the only individuals serving on committee B, the total membership of committee C should be: A. Z and Y B. Z and W C. Y and V D. Y and U E. X and V Ans : A 31. Among the following combinations which could constitute the membership of committee C? A. Y and T B. X and U C. Y, X and W D. W, V and U E. Z, X, U and R Ans : B

(M, N, O and P are all different individuals) I. II. III. M is the daughter of N. N is the son of O O is the father of P.

32. Among the following statements, which is true ? A. O is the uncle of M. B. P and N are brothers C. M is the daughter of P. D. If B is the daughter of N, then M and B are sisters. E. If C is the granddaughter of O, then C and M are sisters. Ans : D

33. Which among the following statements is contradictory to the above premises? A. P is the father of M. B. O has three children. C. M has one brother. D. M is the granddaughter of O. E. Another party C, could be the mother of M. Ans : A 34. If B is the son of N and B has one brother, D, then I. M is the sister of D. II. D and N are brothers. III. O is the grandfather of D. D. I only E. II only F. III only G. I and III only H. I and II only Ans : D The only people to attend a conference were four ship captains and the first mates of three of those captains. The captains were L, M, N and O; the first mates were A, D and G. Each person in turn delivered a report to the assembly as follows: Each of the first mates delivered their report exactly after his or her captain. The first captain to speak was M, and captain N spoke after him. 35. Among the following which is not an appropriate order of delivered reports? A. M, A, N, G, O, L, D B. M, D, N, G, L, O, A C. M, N, A, L, D, O, G D. M, N, A, O, D, L, G E. M, N, G, D, O, L, A Ans : E 36. In case L speaks after A, and A is the third of the first mates to speak, then among the following statements which would be untrue? A. O spoke immediately after G. B. The order of the first four speakers was M, G, N, D. C. O's first mate was present. D. A was the fourth speaker after M. E. The captains spoke in the order M, N, O, L. Ans : D 37. Among the following statements which statement must be true?

A. In case the second speaker was a captain, the seventh speaker was a first mate. B. In case the second speaker was a first mate, the seventh speaker was a captain. C. In case the third speaker was a first mate, the seventh speaker was a captain. D. In case the third speaker was a captain, the seventh speaker was a first mate. E. In case the seventh speaker was a first mate, the first and third speakers were captains. Ans : A 38. In case A spoke immediately after L and immediately before O, and O was not the last speaker, L spoke A. second B. third C. fourth D. fifth E. sixth Ans : C 39. In case G is M's first mate, D could be the person who spoke immediately A. prior to T B. prior to L C. prior to V D. after T E. after V Ans : D 40. In case A is the third of the first mates to speak, and L is the captain whose first mate is not present, which among the following statements must be true? A. A spoke sometime before L. B. D spoke sometime before O. C. L spoke sometime before O. D. O spoke sometime before L. E. O spoke sometime before N. Ans : B 41. Among the following statements, which would make M, D, N, G, L, O, A the only possible sequence of speakers? A. D is M's first mate; G is N's first mate; A is O's first mate. B. D is M's first mate; G is N's first mate; A was the second to speak after L. C. The order of the first four speakers was M, D, N, G. D. The order of the last three speakers was L, O, A.

E. The order in which the captains spoke was M, N, L, O. Ans : B 1. The microbe A causes the contagious disease A. 2. The first symptoms appear after a period of two days since the microbe A enters the body. 3. The microbe A is found in some flies and bees. 4. A fly bit Jack on Monday, February 6. 5. Nick worked with Jack the next day, Tuesday, February 7. There were no other possibilities of exposure to Disease A. 42. In case Jack showed symptoms of Disease A, which of the following statements would be true? I. Jack contracted the Disease A from Nick. II. Jack first noticed symptoms of Disease A on February 8. III. The fly that Jack was bitten by was not a carrier of the microbe A. D. I only E. II only F. III only G. I and II only H. I and III only Ans : B 43. In case Nick displayed symptoms of the disease A, which among the following would be true? . I only A. II only B. III only C. II and III only D. I, II and III Ans : D 44. In case Jack displayed symptoms of Disease A, which would be true? . Jack was also bitten by a fly on February 5. I. Jack was bitten by a mosquito which carried the microbe A. II. Nick contracted Disease A from Jack. 3. I only 4. II only 5. III only 6. I and II only 7. II and III only Ans : D 45. In case Nick displayed the symptoms of Disease A which would be true?

. I. II. 3. 4. 5. 6. 7.

Nick was bitten by a bee on February 6. Nick ate food which contained the microbe A. Nick also worked with Jack on February 6. I only II only III only I and II only I, II and III

Ans : A Questions 46 - 47 The principal evil in today's society is selfishness. Everywhere we see people, who are concerned only with themselves. Personal advancement is the only motivating force in the world today. This does not mean that individuals are not willing to help one another; on the contrary, _________. But, these are only short-term occurrences which ultimately serve our long-term goal of personal gain. 46. To fill in the blank in the above passage, select one of the options from the below mentioned options: 1. we are always trying to undermine others endeavors. 2. my uncle Jeremy used to help me with my homework. 3. no one can be trusted, not even close friends. 4. our yearning for power prevents us from understanding our existential purpose. 5. there are many occasions when we graciously offer our assistance. Ans : E 47. Which among the following options would most strongly contradict the author's attitude towards society? 1. The greatest strength of society is altruism. 2. The forces of good will ultimately triumph over evil. 3. Our short-term actions may ostensibly contradict our long-term goals. 4. We must all learn the art of selfishness. 5. Morality is the bedrock of a growing community. Ans : A 48. It can be proved by an example that our words are devoid of meaning as they cannot be distinguished from their opposites. People think that they are aware of the difference between the meanings of 'bald' and 'having hair' Let us suppose that an average twenty-one year old has X strands of hair on his or her head. We say that such a person is not bald but has hair. But surely one hair less would make no difference, and a person with X - 1 hairs on his or her head would be said to have hair. Suppose we go on like this, decreasing one hair at a time, the result would be the same. Then what difference would there be between someone who has one hair and someone who has none? We call

them both bald. Thus, we cannot make a distinction between the terms 'bald' and 'having hair.' Among the following statements, which statement best counters the argument above? A. The word 'bald' can be translated into other languages. B. A word can have more than one meaning. C. A word such as 'monkey' can be applied to several animals that differ in some respects. D. Words can lack precision without being meaningless. E. People cannot think clearly without using words. Ans : D 49. Virus M helps in controlling the population of gnats; they manage to do this by killing the moth's larvae. Though the virus is always present in the larvae, it is only every six or seven years that the virus seriously decimates the numbers of larvae, greatly reducing the population of the gnats. Scientists are of the opinion that the gnats, usually latent, are activated only when the larvae experience biological stress. In case the above mentioned scientists are correct, it can be inferred that the decimation of gnat populations by the virus M could be most probably activated by the following conditions? A. A shift by the gnats from drought areas to a normal area occupied by them B. The resultant stress from defoliation of trees attacked by the gnats for the second consecutive year. C. Attacks on the larvae by all kinds by parasitic wasps and flies. D. Starvation of the gnat larvae due to over population. E. Spraying of gnat infested areas with laboratory - raised Virus M. Ans : D 50. In a particular code, the digits from 0 to 9 inclusive are each represented by a different letter of the alphabet, the letter always representing the same digit. In case the following sum BOPB +SKB -------CVBQ holds true when it is expressed in digits, which of the following cannot be properly inferred: A. B cannot be 0.

B. C. D. E.

B must be less than 5. Q must be even. O + S must be greater than 8. C must be greater than B by 1.

Ans : B There are three on-off switches on a control panel A, B, and C. They have to be changed from an initial setting to a second setting according to the following conditions : In case only switch A is the switch on in the initial setting , then turn on switch B. In case switches A and B are the only switches on in the initial setting, then turn on switch C. In case all the three switches are on initially setting, then turn off the switch C. For any other initial setting, turn on all switches that are off and turn off all switches, if any, that are on. 51. In case in the initial setting is the switches A and B are on and the switch C is off, then what could be the second setting? A. A on, B on, C on. B. A on, B off, C on. C. A on, B off, C off. D. A off, B on, C off. E. A off, B off, C on. Ans : A 52. In case switch B is the only switch on in the initial setting, what must be the second setting? A. A on, B on, C on. B. A on, B on, C off. C. A on, B off, C on. D. A off, B off, C on. E. A off, B off, C off. Ans : C 53. In case all the three switches are on in the second setting, which among the following could have been the initial setting ? A. A on, B on, C on. B. A on, B on, C off. C. A on, B off, C on. D. A on, B off, C off. E. A off, B on, C off Ans : B 54. In case switch A is off in the second setting, which among the following could have been the initial setting ?

A. B. C. D. E.

A on, B on, C on. A on, B on, C off. A on, B off, C on. A on, B off, C off. A off, B on, C off.

Ans : C 55. In case only switch B is on in the second setting, which among the following could have been the initial setting ? A. A on, B on, C on. B. A on, B off, C on. C. A off, B on, C off. D. A off, B off, C on. E. A off, B off, C off. Ans : B 56. Which among the following initial settings leads to a second setting, where only one switch is off ? A. A on, B on, C off. B. A on, B off, C on. C. A off, B on, C on. D. A off, B on, C off. E. A off, B off, C off. Ans : D A bus has exactly six stops on its route. The bus first stops at stop one and then at stops two, three, four, five, and six respectively. After the bus leaves stop six, the bus turns and returns to stop one and repeats the cycle. The stops are at six building that are, in alphabetical order L, M, N, O, P, and Q. P is the third stop. M is the sixth stop. The stop O is the stop immediately before Q. N is the stop immediately before L. 57. In case N is the fourth stop, which among the following must be the stop immediately before P ? A. O B. Q C. N D. L E. M Ans : B

58. In case L is the second stop, which among the following must be the stop immediately before M ? A. N B. L C. P D. O E. Q Ans : E 59. In case a passenger gets on the bus at O, rides past one of the stops, and gets off at P, which of the following must be true ? A. O is stop one. B. Q is stop three. C. P is stop four. D. N is stop five. E. L is stop six. Ans : A 60. A survey recently conducted revealed that marriage is fattening. The survey found that on an average, women gained 23 pounds and men gained 18 pounds during 13 years of marriage. The answer to which among the following questions would be the most appropriate in evaluating the reasoning presented in the survey ? A. Why is the time period of the survey 13 years, rather than 12 or 14 ? B. Did any of the men surveyed gain less than 18 pounds during the period they were married ? C. How much weight is gained or lost in 13 years by a single people of comparable age to those studied in the survey ? D. When the survey was conducted were the women as active as the men? E. Will the gains seen in the study be retained over the lifetimes of the surveyed persons? Ans : C 61. Classical works occupy a unique and peculiar position. They are imperishable as cultural landmarks, but the views expressed in some of the works are often superseded by newer views. But the classics survive just like ancient castles, as imposing features of the landscape, yet unsuited for habitation unless remodeled. The principal point put forth in the above passage is that classical works. A. Demand our respect and admiration even if their ideas are no longer current. B. Prove to be obstacles in the path of new ideas as they are unduly respected.

C. Should be preserved and treasured as scholarly opinion keeps changing and they will come into fashion again. D. Lose their distinctive features as time passes. E. Are not given due consideration by the younger generation. Ans : A 62. Experts in the field of forestry are now questioning the policy of attempting to extinguish all forest fires instead of allowing them to run their course and die out on their own. The study of forest fires indicates that in the long run, forest fires may in fact, be beneficial to the ecology as a whole. Among the following, which statement would logically follow from one of the observations referred to in the passage above ? A. Most of the forest fires are causes due to natural reasons like lightning rather than as a result of human carelessness. B. Every year the number of people dying as they attempt to fight forest fires is greater than the number of people who would have died or suffered injury if the fires were allowed to run their own course. C. Accumulation of highly flammable dead brushwood and timber, which could lead to massive uncontrollable fires is prevented by frequent small forest fires. D. The animal and plant life destroyed by forest fires seldom represents endangered species. E. Studies indicate that forest fires regularly occurred in the times prior to human occupation of forest areas. Ans : C 63. There is a theory that the dinosaurs inhabiting the earth were destroyed when a huge heavenly body hit the earth. The impact of the heavenly body when it struck the earth threw a mass of pulverized debris into the atmosphere. This dust created a fog, which blocked the sunlight depleting plant life. This resulted in the perishing of the plant-eating dinosaurs; the meat-eating dinosaurs, who preyed on the plant-eaters inturn starved to death. Which among the following statements, would best support the above mentioned theory, if true? A. Enough dust has been sent into the atmosphere by modern volcanic explosions to change the color of sunsets around the world for several years. B. Craters formed by heavenly bodies are plentiful in several areas of the South west where many dinosaur fossils have also been found. C. Studies conducted recently on the bone structure of dinosaurs show that they were very agile and not ponderous awkward animals as popularly believed.

D. The sedimentary rock strata of the earth formed from atmospheric dust at the time the dinosaurs disappeared contains minerals and other trace elements in proportions characteristic of the heavenly bodies. E. Many meat-and plant-eating species of mammals who coexisted with the dinosaurs continue to exist today, although with a slightly modified appearance. Ans : D

Five educational films A, B, C, D, & E are to be shown to a group of students. The films are to be shown in a particular order, which conforms to the following conditions: A must be shown earlier than C. B must be shown earlier than D. E should be the fifth film shown. 64. Which among the following is an acceptable order for showing the educational films ? A. A, C, B, D, E B. A, C, D, E, B C. B, D, C, A, E D. B, D, E, A, C E. E, B, C, A, D Ans : D 65. In case C is shown earlier than E, which among the following will hold true ? A. A is the first film shown. B. B is the second film shown. C. C is the third film shown. D. D is the fifth film shown. E. E is the fourth film shown. Ans : D 66. In case D is to be shown earlier than A, then for which among the following is there exactly one position from first through fifth in which it can be scheduled to be shown ? A. A B. B C. C D. D

E. E Ans : C 67. Which among the following is a pair of films that CANNOT both be shown earlier than E ? A. A and B B. A and D C. B and C D. B and D E. C and D Ans : E 68. In case D and E are shown as far apart from each other as possible, which among the following would be true ? A. A is shown earlier than B. B. B is shown earlier than C. C. C is shown earlier than E. D. D is shown earlier than A. E. E is shown earlier than B. Ans : E 69. In case B, D and E are to be shown one after the other in the given order, the two positions from first to fifth in which A could possibly be shown are A. first and second. B. first and fourth. C. second and third. D. third and fifth. E. fourth and fifth. Ans : B 70. In case exactly one film is shown between A and C, and exactly one film is shown between B and D, which among the following will hold true ? A. B is the film shown between A and C. B. C is the film shown between B and D. C. E is the film shown between A and C. D. D is the last film shown. E. E is the first film shown. Ans : E 71. Authorities concerned with mass transport in metropolitan cities are struggling with deficits. Commuters complain about delays and breakdowns, cuts in service, and fares higher than they are accustomed to paying. For all these reasons and because the price of fuel is still not prohibitive, the number of commuters using public transportation has fallen, adding to the deficits.

Which among the following statements about the relationship between the number of commuters using public transportation and the price of fuel is best supported by the above passage? A. With the rise in the price of fuel, there is a rise in the number of commuters using public transportation. B. Even if the price of fuel rises, the number of commuters using public transportation will continue to decline. C. The number of commuters using public transportation will rise, if the price of fuel rises to a prohibitive level. D. Most of the commuters using public transportation do not use fuel, therefore fluctuations in the price of fuel is unlikely to affect the number of commuters using public transportation. E. The price of fuel is always low enough to make private transportation cheaper than public transportation; therefore fluctuations in the price of fuel is unlikely to affect the number of commuters using public transportation. Ans : C 72. Pharmaceutical firms are now engaged in the production of analogues of endorphins, peptides, which are thought to transmit messages among brain cells, which bring about pain relief. The pharmaceutical firms claim that the analogues, when injected into the bloodstream, will lead to effective and longlasting pain relief by increasing the action of peptides already present in the brain. The claims put forth by the pharmaceutical firms would be considerably weakened if it were true that A. Compared to the other types of neurotransmitters present in the brain, endorphins remain active for a longer time period. B. Certain peptides have been traced in parts besides the brain, like the alimentary canal and the skin. C. It is easier and cheaper to produce analogues of peptides in the laboratory than the peptides themselves. D. Analogues of peptides, which are found naturally in the body are often filtered from the blood before the blood circulates in the brain. E. Endorphins interact chemically both with other naturally occurring peptides and with the brain's other neurotransmitters. Ans : D 73. Quite often it happens that a particular crisis or opportunity induces people to find a practical use for things, which originally had no serious purpose. To exemplify this principle, consider dolls and mannequins, programmed to move and built for the delight of the wealthy in the eighteenth century; these were the forerunners of the modern computer. Similarly, it is almost certain that

pets were the first domesticated animals. Domestication of animals seems to have arisen as an amusement long before it had any practical application. Which among the following, provides another example supporting the above mentioned principle, if true? A. America was discovered as a by-product of the search for ginger, cloves, pepper, and cinnamon. B. Children often imitate adults in their games. C. Till the commercial and scientific potential of the spyglass was recognized and its power of magnification was improved it remained a mere source of diversion. D. Some culture horses were used only for pleasure and not for work, even though in those cultures people had to labor hard in the absence of laboring animals. E. People who made moving dolls and mannequins in the eighteenth century were also clockmakers. Ans : C 74. Ever since the communications-entertainment firms have taken over publishing concerns, the management's new methods have increased the financial profits of commercial publishing. This is done at the price of narrowing the range of available books to the public and by catering to the vulgar tastes of the new buyers of books. There business has boomed ; but in the process the losers are the majority of authors or aspiring writers, and all discriminating readers. In case the above statement is true, which among the following will hold true? A. Profitable business ventures are relatively new in the publishing world. B. The readership which commercial publishing caters to today is different from the readership served in the past. C. The profits resulting from the methods introduced by communicationsentertainment management will encourage literary writers to work against all the odds. D. The limited range of books available to the public is directed toward a more discriminating audience. E. The public is not aware of the trend in the publishing industry, which tends to specialize in publishing blockbuster books. Ans : B 75. It is popularly believed that teachers are more or less indifferent about the microcomputer technology. This assumption is false, or at least dated. A survey recently conducted indicated that 80 percent of the 7,000 surveyed teachers revealed a high level of interest in microcomputers.

Among the following statements which would most damage the above argument if proved to be true? A. There was no attempt made in the survey to ascertain whether the surveyed teachers had any previous exposure to microcomputers. B. Teachers interested in microcomputer technology were more likely to complete and return the questionnaires than others. C. Irrespective of their subject area, their expertise and their teaching experience questionnaires were received by the teachers. D. After the survey results were tabulated there have been many developments in the applications of microcomputer technology. E. A company manufacturing and selling microcomputers conducted the survey. Ans : B Six scientists A, B, C, D, E, and F are to present a paper each at a one-day conference. Three of them will present their papers in the morning session before the lunch break whereas the other three will be presented in the afternoon session. The lectures have to be scheduled in such a way that they comply with the following restrictions: B's should present his paper immediately before C's presentation; their presentations cannot be separated by the lunch break. D must be either the first or the last scientist to present his paper. 76. In case C is to be the fifth scientist to present his paper, then B must be A. first B. second C. third D. fourth E. sixth Ans : D 77. B could be placed for any of the following places in the order of presenters EXCEPT A. first B. second C. third D. fourth E. fifth Ans : C 78. In case F is to present his paper immediately after D presents his paper, C could be scheduled for which of the following places in the order of presenters? A. First B. Second

C. Third D. Fourth E. Fifth Ans : E 79. In case F and E are the fifth and sixth presenters respectively then which of the following must be true? A. A is first in the order of presenters. B. A is third in the order of presenters. C. A is fourth in the order of presenters. D. B is first in the order of presenters. E. C is fourth in the order of presenters. Ans : C In a small inn, one or more of the chefs have to perform duty during dinnereveryday. The chefs are Nicholas, Antonio, and Jeremy. None of them can be assigned to dinner duty two or more days in a row. 80. In case Antonio and Jeremy share the dinner duty thrice over a five-day period, which among the following would be true? A. Nicholas is on dinner duty alone on the first of the five days. B. Nicholas is on dinner duty alone on the second of the five days. C. Nicholas is on dinner duty alone on the third of the five days. D. Antonio and Jeremy share dinner duty on the second of the five days. E. Antonio and Jeremy share dinner duty on the fourth of the five days. Ans : C 81. In case Nicholas and Antonio share dinner duty on Monday of some week, and if Antonio and Jeremy share dinner duty on Thursday of the same week, which of the following would be true for that week? A. Antonio is on dinner duty alone on Tuesday. B. Jeremy is on dinner duty alone on Wednesday. C. Nicholas and Jeremy share dinner duty on Wednesday. D. Nicholas is on duty alone on Tuesday, and Antonio is on breakfast duty alone on Wednesday. E. Jeremy is on breakfast duty alone on Tuesday, and Nicholas is on breakfast duty alone on Wednesday. Ans : E 82. Which among the following could be true of some four-day period? A. On three of the four days, pairs of chefs were on dinner duty. B. On one of the four days, all three chefs were on dinner duty. C. Antonio drew twice as many dinner assignments as did Jeremy. D. Nicholas drew three times as many dinner assignments as did Antonio.

E. Both Nicholas and Antonio drew three times as many dinner assignments as did Jeremy. Ans : C 83. Which among the following statements is the most probable taking into consideration the source and the degree of seriousness with which they are uttered? A. Examiner: "The exam will be taken under the honor system: the professors have the honor, and the students have the system." B. Banker: "The low percentage of Jews in this bank has nothing to do with discrimination. Jews don't seem to be interested in banking." C. Book cover: "This book is sure to become the last word in its field." D. Politician: "My opponent has an impeccable political and personal record." E. Artist: "Einstein's theory of relativity falsifies life." Ans : D 84. Tom: It is likely that Greece will be humiliated in the Cyprus affair. In case she fights Turkey, she will be defeated in battle; whereas if she doesn't fight, letting Turkey win, she will be humiliated, as she would then seem impotent. Sarah: In that case Greece could always call the Soviet Union to her aid. Sarah attacks Tom's argument by: A. B. C. D. E. attacking Tom on a personal level. attacking Tom's method of reasoning siding with Greece advocating passive resistance citing an alternative which was overlooked by Tom.

Ans : E 85. In case Sue sits between Pete and Harry, then Harry sits between Sue and Mike. Harry won't be there unless Sue sits next to Mike. Hence, Sue will not sit between Pete and Harry. Apart from the above mentioned statements, what additional premises are assumed by the author of this argument? Mike sits next to Sue if no one sits between them. If Sue sits between Pete and Harry, then Sue sits between Harry and Pete. If Harry isn't there, then he doesn't sit next to Mike. A. I and II only B. I and III only

C. II and III only D. I, II and III E. None of the above


Ans : B

An increasing number of people prefer to retain their own individuality and their own identity and consequently this has lead to a decline in the marriage rate. 86. Which among the following assumptions are used in the above premises? I. When a person is married, he or she loses his or her own identity and is no longer accountable to himself or herself. II. Married persons do not find contentment as opposed to unmarried people. III. There has been a steady increase in the divorce rate. D. I only E. II only F. III only G. I and II only H. I, II, and III Ans : A 87. Among the following statements, which would weaken the above argument? . The stability resulting from marriage offsets the negative aspects of the dual responsibility of husband and wife. A. Most people are not mature enough to be married. B. Among most married couples the wife wants to have children. C. There are a differing set of values honored by men and women. D. It is advantageous to be single form a tax point of view. Ans : A 88. Which among the following would strengthen the above argument? . Very few people prefer to bring up a family. A. Emotionally divorce is not an easy procedure. B. 700 couples from 1000 surveyed couples complained that they were losing their identity. C. Married people have to make a considerable effort to make the marriage last. D. The financial complications arising from a divorce are becoming decreasingly complicated. Ans : C

89. Emma: Financially the private university is ruined. In case it raises its tuition fees, the number of paying students will come down and if they refrain from raising the fees the university will go bankrupt. Richard: The pay of the teachers and professors could be cut. Richard responds to Emmas argument by . drawing attention to an earlier overlooked alternative. A. attacking her method of reasoning. B. resigning himself to the demise of the private university. C. attacking Emma on a personal level. D. taking the side of the university as against that of the students. Ans : A Questions 90 - 92 Mrs. Green wishes to renovate her cottage. She hires the services of a plumber, a carpenter, a painter, an electrician, and an interior decorator. The renovation is to completed in a period of one working week i.e. Monday to Friday. Every worker will be taking one complete day to do his job. Mrs. Green will allow just one person to work per day. 1. The painter can do his work only after the plumber and the carpenter have completed their jobs. 2. The interior decorator has to complete his job before that of the electrician. 3. The carpenter cannot work on Monday or Tuesday.

90. In case the painter works on Thursday, which among the following alternatives is possible? A. B. C. D. E. The electrician works on Tuesday. The electrician works on Friday. The interior decorator does his work after the painter. The plumber and the painter work on consecutive days. Mrs. Green cannot fit all of the workers into schedule.

Ans : B

91. In case the painter works on Friday, which among the following statements must be untrue? A. The carpenter may work on Wednesday. B. The carpenter and the electrician may work on consecutive days. C. In case the carpenter works on Thursday, the electrician has to work on the previous day i.e. Wednesday. D. The plumber may work before the electrician does. E. The electrician may work on Tuesday. Ans : C 92. Which arrangement among the following is possible? A. The electrician will work on Tuesday and the interior decorator on Friday. B. The painter will work on Wednesday and the plumber on Thursday. C. The carpenter will work on Tuesday and the painter on Friday. D. The painter will work on Monday and the carpenter on Thursday. E. The carpenter will work on Wednesday and the plumber on Thursday. Ans : E 93. According to the laws pertaining to the use of recyclable containers, beverages are sold only in reusable containers. The people who object to such laws point out that the collection of reusable bottles requires gasoline, and washing them requires water; but, it takes less water to wash these containers than it does to make throwaway bottles. The expenditure saved due to the saved water exceeds the cost of the gasoline used to transport empty bottles. The above passage directly answers which of the below mentioned objections to recyclable beverage containers? A. It is likely that consumers will continue to discard even the recyclable containers. B. Consumers will find it an inconvenience to return bottles. C. The extra expense involved in recycling would raise the prices of beverages. D. Recycling would be detrimental to the glass and plastic industries, which produce containers. E. Recycling of containers only partially answers the problem of disposal of solid waste. Ans : C 94. A recently conducted study reveals that the prospects for success in later schooling for 3-5 old children were improved by a particular educational plan. From this it follows that the introduction of similar education plans for all 3-5 year old children will lead to an improvement in their chances for success in later schooling.

Which among the following would weaken the above argument if true? A. Parents of preschool children in the United States get attracted to fads in the educational field, without having a clear idea of the type of early educational plans which would benefit their children. B. The cognitive abilities of children are constantly changing between ages three and five C. The people who conducted the research included quite a few children who had earlier been exposed to another educational enrichment program. D. Many parents are of the view that formal training at an early age takes up the time, which the children could better spend exploring their worlds independently. E. To establish such educational programs at a national level would require extraordinary public expense. Ans : C Questions 95 - 97 Three girls Joan, Rita, and Kim and two boys Tim and Steve are the only dancers in a dance program, which consists of six numbers in this order: One a duet; two a duet; three a solo; four a duet; five a solo; and six a duet. None of the dancers is in two consecutive numbers or in more than two numbers. The first number in which Tim appears is the one that comes before the first number in which Kim appears. The second number in which Tim appears is one that comes after the second number in which Kim appears. 95. Which among the following is a complete and accurate list of those numbers that could be the last one in which Kim performs? A. Three B. Four C. Five D. Three, Four E. Four, Five Ans : E

96. Rita must perform only in duets if A. Kim is in number two B. Kim is in number five C. Tim is in number one D. Tim is in number two

E. Tim is in number six Ans : D 97. In case Steve is in number five, number four must consist of A. two women B. two men C. Tim and a woman D. Rita and a man E. Kim and a man Ans : A

Logical Reasoning
Directions : Each LSAT sample logical reasoning question in this part of the assessment starts with a reading passage containing the information to be used to choose between correct and incorrect logical conclusions. These conclusions are based on the information in the passage. After this reading passage, you are given a lead-in phrase that tells you to choose from among five different responses. These possible responses are generated by correctly or incorrectly applying logical thought to the information in the passage at the beginning of the question. They can be thought of as different ways of completing a sentence that begins with the lead-in phrase. Each reading passage is based on actual Bureau of Labor Statistics documents but is not necessarily a completely accurate representation of BLS work. It is important that you accept every fact in the reading passage as true, when you evaluate the response choices offered. You should use only the information in the passage as the basis for accepting or rejecting any response choices. Be careful not to allow any "facts" that

are not clearly stated in the reading passage, or any outside knowledge you may have of the "facts", to influence your thinking. Following are some LSAT sample logical reasoning questions. 1. Testifying before the Senate committee investigating charges that cigarette manufacturers had manipulated nicotine levels in cigarettes in order to addict consumers to their products, tobacco executives argued that cigarette smoking is not addictive. The primary reason they gave in support of this claim was that cigarette smoking was not regulated by the Federal Drug Administration. For the tobacco executives' argument to be logically correct, which of the following must be assumed? A. Substances that are not addictive are not regulated by ...........the Federal Drug Administration. B. The tobacco executives lied when they claimed that ...........cigarette smoking was not addictive. C. Some addictive substances are not regulated by the ...........Federal Drug Administration. D. There is no scientific proof that cigarette smoking is ...........addictive. E. Substances that are not regulated by the Federal Drug ...........Administration are not addictive. Ans : E 2. People should be held accountable for their own behavior, and if holding people accountable for their own behavior entails capital punishment, then so be it. However, no person should be held accountable for behavior over which he or she had no control. Which of the following is the most logical conclusion of the argument above? A. People should not be held accountable for the ...........behavior of other people. B. People have control over their own behavior. C. People cannot control the behavior of other people. D. Behavior that cannot be controlled should not be ...........punished. E. People have control over behavior that is subject ...........to capital punishment. Ans : B 3. There is clear evidence that the mandated use of safety seats by children under age four has resulted in fewer child fatalities over the past five years. Compared to the five-year period prior to the passage of laws requiring the use of safety seats, fatalities of children under age four have decreased by 30 percent.

Which one of the following, if true, most substantially strengthens the argument above? A. The number of serious automobile accidents involving ...........children under age four has remained steady over the ...........past five years. B. Automobile accidents involving children have decreased ...........sharply over the past five years. C. The use of air bags in automobiles has increased by ...........30 percent over the past five years. D. Most fatal automobile accidents involving children under ...........age four occur in the driveway of their home. E. The number of teenage drivers has increased by 30 ...........percent over the past five years. Ans : A 4. Lycopene, glutathione, and glutamine are powerful antioxidants that neutralize the free radicals that are produced in the body as a result of routine bodily processes. An excess of these free radicals in your system causes rapid aging because they accelerate the rate of cellular damage. Aging is simply the result of this damage. Thus, to slow down aging it is necessary to supplement your diet with these antioxidants on a daily basis. Which of the following, if true, most seriously undermines the author's contention? A. Most persons aren't concerned with the effects of aging ...........until it is too late to do anything. B. Exercise associated with normal daily activities effectively ...........neutralizes and dissipates the free radicals that are ...........produced as a result of routine bodily processes. C. The cost of antioxidants is exorbitantly high and well ...........beyond the budget of most consumers. D. Only overweight people who do not exercise on a daily ...........basis are likely to have an excess of free radicals in their ...........systems. E. Smoking cigarettes is one of the main causes of cellular ...........damage in humans. Ans : B 5. Is it wrong for doctors to lie about their patients' illnesses? Aren't doctors just like any other people we hire to do a job for us? Surely, we would not tolerate not being told the truth about the condition of our automobile from the mechanic we hired to fix it, or the condition of our roof from the carpenter we employed to repair it. Just as these workers would be guilty of violating their good faith contracts with us if they were to do this, doctors who lie to their patients about their illnesses violate these contracts as well, and this is clearly wrong.

The conclusion of the argument is best expressed by which of the following? A. Doctors who lie to their patients about their illnesses ...........violate their good faith contracts with their patients. B. Doctors often lie to their patients about their illnesses. C. Doctors are just hired workers like mechanics and ...........carpenters. D. It is wrong for doctors to lie about their patients' ...........illnesses. E. Doctors, like mechanics and carpenters, enter into good ...........faith contracts with us when we hire them. Ans : D 6. As any economist knows, healthy people pose less of an economic burden to society than unhealthy people. Not surprisingly, then, every dollar our state government spends on prenatal care for undocumented immigrants will save taxpayers of this state three dollars. Which of the following, if true, would best explain why the statistics cited above are not surprising? A. The state's taxpayers pay for prenatal care of all ...........immigrants. B. Babies born in this state to undocumented immigrant ...........parents are entitled to infant care benefits from ...........the state. C. State benefits for prenatal care serve to promote ...........undocumented immigration. D. Babies whose mothers did not receive prenatal care ...........are just as healthy as other babies. E. Pregnant women who do not receive prenatal care are ...........more likely to experience health problems than ...........other pregnant women. Ans : E 7. Beautiful beaches attract people, no doubt about it. Just look at this city's beautiful beaches, which are among the most overcrowded beaches in the state. Which of the following exhibits a pattern of reasoning most similar to the one exhibited in the argument above? A. Moose and bear usually appear at the same drinking ...........hole at the same time of day. Therefore, moose and ...........bear must grow thirsty at about the same time. B. Children who are scolded severely tend to misbehave ...........more often than other children. Hence if a child is ...........not scolded severely that child is less likely to ...........misbehave. C. This software program helps increase the work ...........efficiency of its users. As a result, these users ...........have more free time for other activities.

D. During warm weather my dog suffers from fleas ...........more so than during cooler weather. Therefore, ...........fleas must thrive in a warm environment. E. Pesticides are known to cause anemia in some people. ...........However, most anemic people live in regions where ...........pesticides are not commonly used. Ans : D 8. Our school district should not spend its money on the new Verbal Advantage reading program. After all, our students get all the reading practice they need by studying history and science. The argument above depends on which the following assumptions? A. The Verbal Advantage program would not help the ...........students learn history and science. B. Other reading programs are just as effective but ...........less expensive than the Verbal Advantage program. C. The Verbal Advantage program involves only reading ...........practice. D. Teaching students history and science is more ...........important than teaching them reading skills. E. The students can already read well enough to ...........study history and science Ans : C 9. A study of native born residents in Newland found that two-thirds of the children developed considerable levels of nearsightedness after starting school, while their illiterate parents and grandparents, who had no opportunity for formal schooling, showed no signs of this disability. If the above statements are true, which of the following conclusions is most strongly supported by them? A. Only people who have the opportunity for formal ...........schooling develop nearsightedness. B. People who are illiterate do not suffer from ...........nearsightedness. C. The nearsightedness in the children is caused by the ...........visual stress required by reading and other class work. D. Only literate people are nearsighted. E. One-third of the children are illiterate. Ans : C 10. Newspaper publishers earn their profits primarily from advertising revenue, and potential advertisers are more likely to advertise in newspapers with a wide circulationa large number of subscribers and other readersthan with other newspapers. But the circulation of the newspaper that is currently

the most profitable one in this city has steadily declined during the last two years, while the circulation of one of its competitors has steadily increased. Any of the following, if true, would help explain the apparent discrepancy between the two statements above EXCEPT: A. Advertisers generally switch from the most widely ...........circulated newspaper to another one only when the ...........other one becomes the most widely circulated ...........newspaper instead. B. Advertising rates charged by the most profitable ...........newspaper in the city are significantly higher than ...........those charged by its competitors. C. The most profitable newspaper in the city receives ...........revenue from its subscribers as well from advertisers. D. The circulation of the most profitable newspaper ...........in the city is still greater than than of any of its ...........competitors. E. The number of newspapers competing viably with the ...........most profitable newspaper in the city has increased ...........during the last two years. Ans : E

TEST 2
Directions : Each LSAT sample logical reasoning question in this part of the assessment starts with a reading passage containing the information to be used to choose between correct and incorrect logical conclusions. These conclusions are based on the information in the passage. After this reading passage, you are given a lead-in phrase that tells you to choose from among five different responses. These possible responses are generated by correctly or incorrectly applying logical thought to the information in the passage at the beginning of the question. They can be thought of as different ways of completing a sentence that begins with the lead-in phrase. Each reading passage is based on actual Bureau of Labor Statistics documents but is not necessarily a completely accurate representation of BLS work. It is important that you accept every fact in the reading passage as true, when you evaluate the response choices offered. You should use only the information in the passage as the basis for accepting or rejecting any response choices. Be careful not to allow any "facts" that are not clearly stated in the reading passage, or any outside knowledge you may have of the "facts", to influence your thinking. Following are some LSAT sample logical reasoning questions. 1. Testifying before the Senate committee investigating charges that cigarette manufacturers had manipulated nicotine levels in cigarettes in order to addict

consumers to their products, tobacco executives argued that cigarette smoking is not addictive. The primary reason they gave in support of this claim was that cigarette smoking was not regulated by the Federal Drug Administration. For the tobacco executives' argument to be logically correct, which of the following must be assumed? A. Substances that are not addictive are not regulated by ...........the Federal Drug Administration. B. The tobacco executives lied when they claimed that ...........cigarette smoking was not addictive. C. Some addictive substances are not regulated by the ...........Federal Drug Administration. D. There is no scientific proof that cigarette smoking is ...........addictive. E. Substances that are not regulated by the Federal Drug ...........Administration are not addictive. Ans : E 2. People should be held accountable for their own behavior, and if holding people accountable for their own behavior entails capital punishment, then so be it. However, no person should be held accountable for behavior over which he or she had no control. Which of the following is the most logical conclusion of the argument above? A. People should not be held accountable for the ...........behavior of other people. B. People have control over their own behavior. C. People cannot control the behavior of other people. D. Behavior that cannot be controlled should not be ...........punished. E. People have control over behavior that is subject ...........to capital punishment. Ans : B 3. There is clear evidence that the mandated use of safety seats by children under age four has resulted in fewer child fatalities over the past five years. Compared to the five-year period prior to the passage of laws requiring the use of safety seats, fatalities of children under age four have decreased by 30 percent. Which one of the following, if true, most substantially strengthens the argument above? A. The number of serious automobile accidents involving ...........children under age four has remained steady over the ...........past five years. B. Automobile accidents involving children have decreased ...........sharply over the past five years.

C. The use of air bags in automobiles has increased by ...........30 percent over the past five years. D. Most fatal automobile accidents involving children under ...........age four occur in the driveway of their home. E. The number of teenage drivers has increased by 30 ...........percent over the past five years. Ans : A 4. Lycopene, glutathione, and glutamine are powerful antioxidants that neutralize the free radicals that are produced in the body as a result of routine bodily processes. An excess of these free radicals in your system causes rapid aging because they accelerate the rate of cellular damage. Aging is simply the result of this damage. Thus, to slow down aging it is necessary to supplement your diet with these antioxidants on a daily basis. Which of the following, if true, most seriously undermines the author's contention? A. Most persons aren't concerned with the effects of aging ...........until it is too late to do anything. B. Exercise associated with normal daily activities effectively ...........neutralizes and dissipates the free radicals that are ...........produced as a result of routine bodily processes. C. The cost of antioxidants is exorbitantly high and well ...........beyond the budget of most consumers. D. Only overweight people who do not exercise on a daily ...........basis are likely to have an excess of free radicals in their ...........systems. E. Smoking cigarettes is one of the main causes of cellular ...........damage in humans. Ans : B 5. Is it wrong for doctors to lie about their patients' illnesses? Aren't doctors just like any other people we hire to do a job for us? Surely, we would not tolerate not being told the truth about the condition of our automobile from the mechanic we hired to fix it, or the condition of our roof from the carpenter we employed to repair it. Just as these workers would be guilty of violating their good faith contracts with us if they were to do this, doctors who lie to their patients about their illnesses violate these contracts as well, and this is clearly wrong. The conclusion of the argument is best expressed by which of the following? A. Doctors who lie to their patients about their illnesses ...........violate their good faith contracts with their patients. B. Doctors often lie to their patients about their illnesses. C. Doctors are just hired workers like mechanics and ...........carpenters. D. It is wrong for doctors to lie about their patients' ...........illnesses.

E. Doctors, like mechanics and carpenters, enter into good ...........faith contracts with us when we hire them. Ans : D 6. As any economist knows, healthy people pose less of an economic burden to society than unhealthy people. Not surprisingly, then, every dollar our state government spends on prenatal care for undocumented immigrants will save taxpayers of this state three dollars. Which of the following, if true, would best explain why the statistics cited above are not surprising? A. The state's taxpayers pay for prenatal care of all ...........immigrants. B. Babies born in this state to undocumented immigrant ...........parents are entitled to infant care benefits from ...........the state. C. State benefits for prenatal care serve to promote ...........undocumented immigration. D. Babies whose mothers did not receive prenatal care ...........are just as healthy as other babies. E. Pregnant women who do not receive prenatal care are ...........more likely to experience health problems than ...........other pregnant women. Ans : E 7. Beautiful beaches attract people, no doubt about it. Just look at this city's beautiful beaches, which are among the most overcrowded beaches in the state. Which of the following exhibits a pattern of reasoning most similar to the one exhibited in the argument above? A. Moose and bear usually appear at the same drinking ...........hole at the same time of day. Therefore, moose and ...........bear must grow thirsty at about the same time. B. Children who are scolded severely tend to misbehave ...........more often than other children. Hence if a child is ...........not scolded severely that child is less likely to ...........misbehave. C. This software program helps increase the work ...........efficiency of its users. As a result, these users ...........have more free time for other activities. D. During warm weather my dog suffers from fleas ...........more so than during cooler weather. Therefore, ...........fleas must thrive in a warm environment. E. Pesticides are known to cause anemia in some people. ...........However, most anemic people live in regions where ...........pesticides are not commonly used.

Ans : D 8. Our school district should not spend its money on the new Verbal Advantage reading program. After all, our students get all the reading practice they need by studying history and science. The argument above depends on which the following assumptions? A. The Verbal Advantage program would not help the ...........students learn history and science. B. Other reading programs are just as effective but ...........less expensive than the Verbal Advantage program. C. The Verbal Advantage program involves only reading ...........practice. D. Teaching students history and science is more ...........important than teaching them reading skills. E. The students can already read well enough to ...........study history and science Ans : C 9. A study of native born residents in Newland found that two-thirds of the children developed considerable levels of nearsightedness after starting school, while their illiterate parents and grandparents, who had no opportunity for formal schooling, showed no signs of this disability. If the above statements are true, which of the following conclusions is most strongly supported by them? A. Only people who have the opportunity for formal ...........schooling develop nearsightedness. B. People who are illiterate do not suffer from ...........nearsightedness. C. The nearsightedness in the children is caused by the ...........visual stress required by reading and other class work. D. Only literate people are nearsighted. E. One-third of the children are illiterate. Ans : C 10. Newspaper publishers earn their profits primarily from advertising revenue, and potential advertisers are more likely to advertise in newspapers with a wide circulationa large number of subscribers and other readersthan with other newspapers. But the circulation of the newspaper that is currently the most profitable one in this city has steadily declined during the last two years, while the circulation of one of its competitors has steadily increased. Any of the following, if true, would help explain the apparent discrepancy between the two statements above EXCEPT:

A. Advertisers generally switch from the most widely ...........circulated newspaper to another one only when the ...........other one becomes the most widely circulated ...........newspaper instead. B. Advertising rates charged by the most profitable ...........newspaper in the city are significantly higher than ...........those charged by its competitors. C. The most profitable newspaper in the city receives ...........revenue from its subscribers as well from advertisers. D. The circulation of the most profitable newspaper ...........in the city is still greater than than of any of its ...........competitors. E. The number of newspapers competing viably with the ...........most profitable newspaper in the city has increased ...........during the last two years. Ans : E

11. Although most of the fastest growing jobs in today's economy will require a college degree, many of the new jobs being created-from home health aide to desktop publisher-require knowledge other than that gained from earning a degree. For workers in those jobs, good basic skills in reading, communication, and mathematics play an important role in getting a job and developing a career. From the information given above it can be validly concluded that, in today's economy, A. skills in reading, communication, and mathematics play an important role in developing a career as a desktop publisher B. the majority of the new jobs being created require knowledge other than that gained from earning a college degree C. a job as a home health aide will rely more on communication skills than on basic skills in reading and mathematics D. if a job is one of the fastest growing jobs, it will require a college degree E. desktop publisher jobs and home health aide jobs are not among the fastest growing jobs Ans : A 12. According to the National Agricultural Aviation Society (NAAS), without the use of crop protection products to control insects, weeds, and diseases, crop yields per acre will drop by more than 50 percent. The first aerial application of insecticide occurred in 1921, and it was a huge success. By contrast, in today's economy all aircraft that are classified as aerial applicators do more than just apply insecticide; today, they also spread seed and apply fertilizer. From the information given above it CANNOT be validly concluded that

A. in today's economy, if an aerial applicator is used, then it will be able to spread seed and to apply fertilizer B. according to the NAAS, if crop yields per acre never drop by more than 50 percent, then crop protection products have been used to control insects, weeds, and diseases C. in today's economy, any aircraft that cannot be used to apply fertilizer cannot be classified as an aerial applicator D. in 1921, if an aircraft was used for the application of insecticide, then it was not also used to spread seed E. according to the NAAS, if crop yields per acre drop by more than 50 percent, then crop protection products have not been used to control insects, weeds, and diseases. Ans : E 13. No national productivity measures are available for underground industries that may exist but remain unreported. On the other hand, at least some industries that are run entirely by self-employed industrialists are included in national productivity measures. From the information given above, it can be validly concluded that A. there are at least some industries run entirely by self-employed industrialists that are underground industries B. no industries that are run entirely by self-employed industrialists operate underground C. there are at least some industries other than those run entirely by selfemployed industrialists that are underground industries D. there are at least some industries run entirely by self-employed industrialists that are not underground industries E. there are at least some underground industries for which national productivity measures are available Ans : D 14. Lou observes that if flight 409 is canceled, then the manager could not possibly arrive in time for the meeting. But the flight was not canceled. Therefore, Lou concludes, the manager will certainly be on time. Evelyn replies that even if Lou's premises are true, his argument is fallacious. And therefore, she adds, the manager will not arrive on time after all. Which of the following is the strongest thing that we can properly say about this discussion? A. Evelyn is mistaken in thinking Lou's argument to be fallacious, and so her own conclusion is unwarranted. B. Evelyn is right about Lou's argument, but nevertheless her own conclusion is unwarranted.

C. Since Evelyn is right about Lou's argument, her own conclusion is well supported. D. Since Evelyn is mistaken about Lou's argument, her own conclusion must be false. E. Evelyn is right about Lou's argument, but nevertheless her own conclusion is false. Ans : B 15. Sally has never received a violation from the Federal Aviation Administration during her 16-year flying career. Sally must be a great pilot. Which of the following can be said about the reasoning above? A. B. C. D. E. The definitions of the terms create ambiguity. The argument uses circular reasoning. The argument works by analogy. The argument is built upon hidden assumptions. This is an example of an argument that is directed against the source of the claim rather than the claim itself.

Ans : D 16. The Japanese economic model created strong domestic industries through subsidies from its Ministry of Trade and by closing off competitive foreign firms to its domestic market. This strategy promised to help economic growth by incubating domestic industries. New Japanese industries could count on a known local demand and would be protected from competition by tariffs and other barriers. The program could reduce the amount of imports and therefore improve the nation's balance of trade. Which of the following, based on the passage above, is a weakness in this economic strategy? A. A protectionist policy will create animosity among other nations. B. Fast growth of small industries will create a class of millionaires and increase the inequality of income. C. Subsidies and import constraints keep domestic prices high and impose a burden on consumers. D. Quotas are more regressive than tariffs. E. The demand for the products made by the incubated industries would not be known. Ans : C 17. Historically, famines have generally been followed by periods of rising wages, because when a labor force is diminished, workers are more valuable in accordance with the law of supply and demand. The Irish potato famine of the 1840s is an exception; it resulted in the death or emigration of half of Ireland's

population, but there was no significant rise in the average wages in Ireland in the following decade. Which one of the following, if true, would LEAST contribute to an explanation of the exception to the generalization? A. Improved medical care reduced the mortality rate among able-bodied adults in the decade following the famine to below prefamine levels. B. Eviction policies of the landowners in Ireland were designed to force emigration of the elderly and infirm, who could not work, and to retain a high percentage of able-bodied workers. C. Advances in technology increased the efficiency of industry and agriculture, and so allowed maintenance of economic output with less demand for labor. D. The birth rate increased during the decade following the famine, and this compensated for much of the loss of population that was due to the famine. E. England, which had political control of Ireland, legislated artificially low wages to provide English-owned industry and agriculture in Ireland with cheap labor. Ans : D 18. Cars are safer than planes. Fifty percent of plane accidents result in death, while only one percent of car accidents result in death. Which of the following, if true, would most seriously weaken the argument above? A. Planes are inspected more often than cars. B. The number of car accidents is several hundred thousand times higher than the number of plane accidents. C. Pilots never fly under the influence of alcohol, while car drivers often do. D. Plane accidents are usually the fault of air traffic controllers, not pilots. E. Planes carry more passengers than cars do. Ans : B 19. The body of anyone infected by virus X will, after a week, produce antibodies to fight the virus; the antibodies will increase in number for the next year or so. There is now a test that reliably indicates how many antibodies are present in a person's body. If positive, this test can be used during the first year of infection to estimate to within a month how long that person has had the virus. Which one of the following conclusions is best supported by the statements above? A. Antibodies increase in number only until they have defeated the virus.

B. Without the test for antibodies, there is no way of establishing whether a person has virus X. C. Antibodies are produced only for viral infections that cannot be fought by any other body defenses. D. If a person remains infected by virus X indefinitely, there is no limit to the number of antibodies that can be present in the person's body. E. Anyone infected by virus X will for a time fail to exhibit infection if tested by the antibody test. Ans : E 20. Ever since I arrived at the college last week, I've been shocked by the poor behavior of the students. The student population is completely lacking in proper social skills. Which of the following, if true, would weaken the above conclusion? A. B. C. D. E. Students who are away from their parents often exhibit rude behavior. The college numbers over 50,000 students. The narrator is a student and has interacted with many students. Social skills should not be expected of college students. The narrator was reluctant to stay at the college.

Ans : B 21. A study of a math program implemented in several pre-schools indicates that children who received the specialized Math Plus math education between the ages three and five had significantly higher math scores in 3rd and 4th grade than their classmates who did not receive this instruction. The proponents of the math argue that the introduction of this program for all children age three to five will significantly improve their chances for success in school. Which of the following, if true, would most seriously weaken the above argument? A. Most parents send their children to preschool for social development and do not have a clear idea about what types of education they want for their children. B. Cognitive abilities of 3- to 5-year-old children are constantly changing. C. The children in the pre-schools that were studied had previously been exposed to another math enrichment program. D. Children are not really interested in enrichment programs in preschool. E. The cost factor needs to be specified and established before a large scale program can be undertaken. Ans : C 22. The symptoms of mental disorders are behavioral, cognitive, or emotional problems. Some patients with mental disorders can be effectively treated with

psychotherapy, but it is now known that in some patients' mental disorders result from chemical imbalances affecting the brain. Thus, these patients can be effectively treated only with medication that will reduce or correct the imbalance. The argument depends on assuming which one of the following? A. Treatment by psychotherapy can produce no effective reduction in or correction of chemical imbalances that cause mental disorders. B. Treatment with medication always shows faster results for patients with mental disorders than does treatment with psychotherapy C. Most mental disorders are not the result of chemical imbalances affecting the brain. D. Medication is always more effective in treating patients with mental disorders than is psychotherapy. E. Treatment with psychotherapy has no effect on mental disorders other than a reduction of the symptoms. Ans : A 23. Dear Editor: I feel obliged to comment on the unfair review you published last week written by Robert Duxbury. Your readers should know that Mr. Duxbury recently published his own book that covered the same topic as my book, which you asked him to review. It is regrettable that Mr. Duxbury should feel the need to belittle a competing work in the hope of elevating his own book. The author of the letter above makes her point by employing which method of argument? A. Attacking the motives of the author of the unfavorable review. B. Attacking the book on the same topic written by the author of the review. C. Contrasting her own book with that written by the author of the review. D. Questioning the judgment of the author of the unfavorable review. E. Stating that her book should not have been reviewed by the author of a competing work. Ans : A 24. The government of Zunimagua has refused to schedule free elections, release political prisoners, or restore freedom of speech; therefore, no more financial aid from the United States should be provided to Zunimagua. Which of the following is an assumption made in the argument above? A. Withdrawal of U.S. aid from Zunimagua will force a change in the policies of its government. B. The people of Zunimagua would be better off if their present despotic government were overthrown.

C. The government of Zunimagua is dependent on continued U.S. aid for its existence. D. U.S. aid should be given only to countries willing to adopt policies in line with U.S. interests and goals. E. U.S. aid should be withdrawn from any country that refuses to operate its government along democratic lines. Ans : E 25. Many people argue that the death penalty deters murder. However, the notorious killer Ned Grandy deliberately moved to a state that imposes the death penalty just before embarking on a series of ferocious murders. Thus, it seems clear that the existence of the death penalty does not serve as a deterrent to murder. The argument above may best be characterized as: A. B. C. D. E. an appeal to emotion. a flawed analogy. a general conclusion based on a specific example. circular reasoning. an application of a general principle to a specific example.

Ans : C 26. Steve and JoAnne are both members of a certain club, though they are not speaking to each other and refuse to work with each other. Cecily, the club president, is appointing members to the fundraising committee, but she has resolved that she will not appoint anyone without his or her explicit consent. Steve tells Cecily, "I will not consent to appointment on that committee unless I know whether JoAnne is to be a member of it." And JoAnne says, "I will not consent to be a member of that committee unless I know whether Steve will be appointed to it." If all three of these people stick by these resolutions, then: A. Neither of them can be appointed to the committee. B. The situation described in the scenario cannot arise, because it is inherently incoherent. C. They must either both be appointed or both be left out. D. The committee may finally have one of them, both of them, or neither of them as members. E. Either one of them can be appointed, but not both. Ans : E 27. Russia's aggressive fishing in the prime fishing grounds of the Northern Pacific has led to a sharp decline in the populations of many fish and a general increase in the retail price of fish. This same pattern has occurred with far too

many of our scarce vital natural resources, resulting in high prices for many products. It is likely then, that fish prices will continue to rise in the near future. In making the argument above, the author relies on all of the following assumptions EXCEPT: A. The scarcity of fish is a determining factor in its price. B. The decline in the number of fish available will result in higher prices for fish in stores. C. There will not be any substantial decrease in other costs involved in the fishing process that could keep the price of fish from increasing. D. Fish populations will not recover in the near future. E. Fishing practices can substantially influence the demand for fish. Ans : E 28. During the past year, Boz Corporation, a cigarette manufacturer, has engaged in a "corporate image" advertising campaign. One executive now urges that the advertising be extended for another year because profits have increased by 29% over the previous year. Another executive, however, is skeptical. She observes that the increases are typical for the industry over the past year, although none of their competitors have used corporate image advertising. The most accurate way of summarizing the second executive's point would be: A. She argues that the effect may not really be due to its supposed cause because there has not been a sufficient lapse of time between the cause and the effect. B. She argues that the assignment of a cause for this effect is premature, because there is as yet no well-established theory of such interactions. C. She argues that corporate image advertising is unprofitable, since it has evidently benefited competitors as much as the corporation that paid for it. D. She knows that effective advertising requires a constant influx of new ideas and approaches, and she argues that one year of corporate image advertising is enough for awhile. E. She argues that the effect may not be due to its alleged cause since the same effect is found elsewhere without that cause. Ans : E 29. The senate candidate expressed outrage that few judges have any background in technology, yet they try to resolve cases involving high tech companies. He stated that not one federal judge has a degree or any experience in computer technology. A promising response to this concern, arguing that things are not as bad as they might seem, could involve which of the following claims?

A. Most of the public policy questions in this area are really about the morality and the value of scientific and technological developments. They do not require much technical understanding beyond that of a layperson. B. Computer scientists, by and large, have little interest in politics and public policy. It would be difficult to find scientists with the degree of commitment required for a serious contribution to the judicial system. C. There is a lack of people who are qualified in both technical and legal areas of expertise. D. There is very little opportunity for, and indeed little need for, technical expertise in the judicial branch. There is therefore almost no way for a technical specialist to rise through the ranks to a top-level position in government. E. The rewards of a life as a judge, in terms of both money and prestige, are not high enough to attract top-flight technical experts to this area. Ans : A 30. There has been a sharp increase in the subscription prices of many professional and scholarly journals in the past seven years. Many publishers ascribe the necessity for these increases to the easy availability of photocopying facilities, which enable people simply to copy the articles they want rather than buying the journal. Which of the following, if it is true, would make this explanation more plausible? A. The great majority of professional and scholarly journals have a massive backlog of papers awaiting publication. B. Over the past five years there has been a substantial decline in the number of individual subscriptions to professional and scholarly journals, while library subscriptions have remained fairly stable. C. In the five years immediately preceding the price surge, there was a substantial decline in the number of individual subscriptions to professional and scholarly journals, while library subscriptions remained fairly stable. D. Many libraries have recently begun cutting back on subscriptions to professional and scholarly journals. E. In almost every field, several new professional and scholarly journals have begun publication in the past few years. Ans : C 31. Smoking in bed has long been the main cause of home fires. Despite a significant decline in cigarette smoking in the last two decades, there has been no comparable decline in the number of people killed in home fires. Each one of the following statements, if true over the last two decades, helps to resolve the apparent discrepancy above EXCEPT:

A. Compared to other types of home fires, home fires caused by smoking in bed usually cause relatively little damage before they are extinguished. B. Home fires caused by smoking in bed often break out after the home's occupants have fallen asleep. C. Smokers who smoke in bed tend to be heavy smokers who are less likely to quit smoking than are smokers who do not smoke in bed. D. An increasing number of people have been killed in home fires that started in the kitchen. E. Population densities have increased, with the result that one home fire can cause more deaths than in previous decades. Ans : B 32. Mrs. Mason is gifted with psychic powers that enable her to foretell future events. In the past, Mrs. Mason has predicted such actual events as the election of President Clinton, the stock market crash of 1987, and the St. Louis Cardinals' 1982 World Series victory. These are just a few of Mrs. Mason's accurate predictions. The answer to which of the following questions would be most useful in evaluating the strength of the argument above? A. What percentage of Mrs. Mason's predictions has come true? B. Could the election of President Reagan have been predicted without the help of psychic powers? C. What is the actual mechanism by which Mrs. Mason's psychic powers are supposed to operate? D. How long before the events in question did Mrs. Mason make her accurate predictions? E. Do most scientists accept the idea that the power to predict the future through psychic means really exists? Ans : A 33. An ingredient in coffee, known as RTC, has been found to inactivate common cold viruses in experiments. In previous experiments, researchers found that inactivated common cold viruses can convert healthy cells into cancer cells. It can be concluded that the use of coffee can cause cancer. Which one of the following, if true, most seriously weakens the argument? A. Several teams of scientists performed the various experiments, and all of the teams had similar results. B. The carcinogenic effect of RTC could be neutralized by the other ingredients found in coffee. C. When RTC kills common cold viruses it weakens the immune system, and it might thus diminish the body's ability to fight other viruses, including viruses linked to cancers.

D. If chemists modify the structure of RTC, RTC can be safely incorporated into medications to prevent the common cold. E. To lessen the undesirable side effects of chemotherapy, the use of coffee has been recommended for cancer patients who are free of the common cold virus. Ans : B 34. Jack Bygrave is an executive at a major South African diamond company that produces 2% of the world's total annual diamond production. The CFO is anxious to maximize revenues and increase sales. Bygrave, however, believes that increased production would only drive down the world price of diamonds and lower revenues. Which of the following represents the logical flaw in Bygrave's reasoning? A. Jack connects the price of unrefined diamonds and the price of jewelry-quality diamonds. B. He assumes that production goals are similar to financial goals. C. He assumes that the supply produced by a single company can significantly alter the aggregate supply for the market. D. He assumes that seasonal and long term supply are proportional. E. He correlates long-term and short-term demand. Ans : C 35. The crux of creativity resides in the ability to manufacture variations on a theme. If we look at the history of science, for instance, we see that every idea is built upon a thousand related ideas. Careful analysis leads us to understand that what we choose to call a new theme or a new discovery is itself always and without exception some sort of variation, on a deep level, of previous themes. If all of the statements in the passage are true, each of the following must also be true EXCEPT: A. A lack of ability to manufacture a variation on a previous theme connotes a lack of creativity B. No scientific idea is entirely independent of all other ideas. C. Careful analysis of a specific variation can reveal previous themes of which it is a variation. D. All great scientific discoverers have been able to manufacture a variation on a theme. E. Some new scientific discoveries do not represent, on a deep level, a variation on previous themes. Ans : E

36. Studies of fatal automobile accidents reveal that, in the majority of cases in which one occupant of an automobile is killed while another survives, it is the passenger, not the driver, who is killed. It is ironic that the innocent passenger should suffer for the driver's carelessness, while the driver often suffers only minor injuries or none at all. Which of the following is an assumption underlying the reasoning in the passage above? A. In most fatal automobile accidents, the driver of a car in which an occupant is killed is at fault. B. Drivers of automobiles are rarely killed in auto accidents. C. Most deaths in fatal automobile accidents are suffered by occupants of cars rather than by pedestrians. D. Auto safety experts should increase their efforts to provide protection for those in the passenger seats of automobiles. E. Automobile passengers sometimes play a contributing role in causing auto accidents. Ans : A 37. The editors of Business Today magazine conducted a poll of its readers regarding the proposed increase in the rate of income tax paid on profits from the sale of stocks. More than 60% of the readers opposed the proposed tax. The editors announced that the majority of Americans opposed any increase in the tax on profits from stock sales. Which one of the following statements, if true, would most weaken the editor's conclusion? A. Some readers of Business Today magazine are citizens of countries other than the United States. B. Decisions concerning the income tax laws are made by the Congress rather than directly by the people. C. Most of those who earn profits from stock sales are wealthy and can afford to pay higher taxes. D. The viewpoints of the vast majority of the readers of Business Today magazine differ from the views of most Americans. E. Not all readers of Business Today magazine responded to the editors' poll. Ans : D 38. Several movie versions of Charles Dickens' Tale of Two Cities have been made. The original movie version made in 1939 is the best because it is closest in spirit to the original novel. An underlying assumption of the argument above is that a movie based on a novel should:

A. reflect the director's original interpretation of the main themes of the novel. B. accurately depict the time and place in which the novel is set. C. feature actors and actresses who closely resemble the characters in the novel both in body and spirit. D. faithfully render the details of the plot from the narrator's point of view. E. capture the true meaning and intention of the novel. Ans : E

Reading Comprehension
Directions: Each LSAT sample reading passage in this section is followed by questions based on the content of the reading passage. Read the passage carefully and chose the best answer to each question. The questions are to be answered on the basis of what is stated or implied in the passage. 1. But man is not destined to vanish. He can be killed, but he cannot be destroyed, because his soul is deathless and his spirit is irrepressible. Therefore, though the situation seems dark in the context of the confrontation between the superpowers, the silver lining is provided by amazing phenomenon that the very nations which have spent incalculable resources and energy for the production of deadly weapons are desperately trying to find out how they might never be used. They threaten each other, intimidate each other and go to the brink, but before the total hour arrives they withdraw from the brink. 1. The main point from the author's view is that A. Man's soul and spirit can not be destroyed by superpowers. B. Man's destiny is not fully clear or visible.

C. Man's soul and spirit are immortal. D. Man's safety is assured by the delicate balance of power in terms of nuclear weapons. E. Human society will survive despite the serious threat of total annihilation. Ans : E 2. The phrase 'Go to the brink' in the passage means A. Retreating from extreme danger. B. Declare war on each other. C. Advancing to the stage of war but not engaging in it. D. Negotiate for peace. E. Commit suicide. Ans : C 3. In the author's opinion A. Huge stockpiles of destructive weapons have so far saved mankind from a catastrophe. B. Superpowers have at last realized the need for abandoning the production of lethal weapons. C. Mankind is heading towards complete destruction. D. Nations in possession of huge stockpiles of lethal weapons are trying hard to avoid actual conflict. E. There is a Silverlining over the production of deadly weapons. Ans : D 4. 'Irrepressible' in the second line means A. incompatible B. strong C. oppressive D. unrestrainable E. unspirited Ans : D 5. A suitable title for the above passage is A. Destruction of mankind is in evitable. B. Man's desire to survive inhibits use of deadly weapons. C. Mounting cost of modern weapons. D. Threats and intimidation between super powers. E. Cowardly retreat by man Ans : B

2. Disequilibrium at the interface of water and air is a factor on which the transfer of heat and water vapor from the ocean to the air depends. The air within about a millimeter of the water is almost saturated with water vapor and the temperature of the air is close to that of the surface water. Irrespective of how small these differences might be, they are crucial, and the disequilibrium is maintained by air near the surface mixing with air higher up, which is typically appreciably cooler and lower in water vapor content. The turbulence, which takes its energy from the wind mixes the air. As the speed of wind increases, so does the turbulence, and consequently the rate of heat and moisture transfer. We can arrive at a detailed understanding of this phenomenon after further study. The transfer of momentum from wind to water, which occurs when waves are formed is an interacting-and complicated phenomenon. When waves are made by the wind, it transfers important amounts of energy-energy, which is consequently not available for the production of turbulence. 1. This passage principally intends to: A. resolve a controversy B. attempt a description of a phenomenon C. sketch a theory D. reinforce certain research findings E. tabulate various observations Ans : B 2. The wind over the ocean usually does which of the following according to the given passage? I. Leads to cool, dry air coming in proximity with the ocean surface. II. Maintains a steady rate of heat and moisture transfer between the ocean and the air. III. Results in frequent changes in the ocean surface temperature. A. I only B. II only C. I and II only D. II and III only E. I, II, and III Ans : A 3. According to the author the present knowledge regarding heat and moisture transfer from the ocean to air as A. revolutionary B. inconsequential C. outdated D. derivative E. incomplete Ans : E

4. According to the given passage, in case the wind was to decrease until there was no wind at all, which of the following would occur? A. The air, which is closest to the ocean surface would get saturated with water vapor. B. The water would be cooler than the air closest to the ocean surface. C. There would be a decrease in the amount of moisture in the air closest to the ocean surface. D. There would be an increase in the rate of heat and moisture transfer. E. The temperature of the air closest to the ocean and that of the air higher up would be the same. Ans : A

3. The Food and Drug Administration has formulated certain severe restrictions regarding the use of antibiotics, which are used to promote the health and growth of meat animals. Though the different types of medicines mixed with the fodder of the animals kills many microorganisms, it also encourages the appearance of bacterial strains, which are resistant to anti-infective drugs. It has already been observed that penicillin and the tetracyclines are not as effective therapeutically as they once used to be. This resistance to drugs is chiefly caused due to tiny circlets of genes, called plasmids, which are transferable between different species of bacteria. These plasmids are also one of the two kinds of vehicles on which molecular biologists depend on while performing gene transplant experiments. Existing guidelines also forbid the use of plasmids, which bear genes for resistance to antibiotics, in the laboratories. Though congressional dabate goes on as to whether these restrictions need to be toughened with reference to scientists in their laboratories, almost no congressional attention is being paid to an ill advised agricultural practice, which produces deleterious effects. 1. In the present passage, the author's primary concern is with: A. The discovery of methods, which eliminate harmful microorganisms without generating drug-resistant bacteria. B. Attempting an explanation of the reasons for congressional inaction about the regulation of gene transplant experiments. C. Portraying a problematic agricultural practice and its serious genetic consequences D. The verification of the therapeutic ineffectiveness of anti-infective drugs

E. Evaluation of the recently proposed restrictions, which are intended to promote the growth of meat animals. Ans : C 2. As inferred from the above passage, the mutual transfer of plasmids between different bacteria can result in which of the following? A. Microorganisms, which have an in-built resistance to drugs B. Therapeutically useful circlets of genes C. Penicillin like anti-infective drugs D. Viruses used by molecular biologists E. Carriers for performing gene transplant experiments. Ans : A 3. According to the above passage the author believes that those who favor the stiffening of restrictions on gene transplant research should logically also. A. Approve and aid experiments with any plasmids except those, which bear genes for antibiotic resistance. B. Inquire regarding the addition of anti-infective drugs to livestock feeds C. Oppose the using of penicillin and tetracyclines in order to kill microorganisms D. Agree to the development of meatier live-stock through the use of antibiotics E. Approve of congressional debate and discussion regarding science and health issues. Ans : B 4. The attitude the author has with reference to the development of bacterial strains that render antibiotic drugs in effective can best be described as A. indifferent B. perplexed C. pretentious D. insincere E. apprehensive Ans : E

4. Roger Rosenblatt's book Black Fiction, manages to alter the approach taken in many previous studies by making an attempt to apply literary rather than sociopolitical criteria to its subject. Rosenblatt points out that criticism of Black writing has very often served as a pretext for an expounding on Black history. The recent work of Addison Gayle's passes a judgement on the value of Black fiction by clearly political standards, rating each work according to the ideas of Black identity, which it propounds.

Though fiction results from political circumstances, its author react not in ideological ways to those circumstances, and talking about novels and stories primarily as instruments of ideology circumvents much of the fictional enterprise. Affinities and connections are revealed in the works of Black fiction in Rosenblatt's literary analysis; these affinities and connections have been overlooked and ignored by solely political studies. The writing of acceptable criticism of Black fiction, however, presumes giving satisfactory answers to a quite a few questions. The most important of all, is there a sufficient reason, apart from the racial identity of the authors, for the grouping together of Black authors? Secondly, what is the distinction of Black fiction from other modern fiction with which it is largely contemporaneous? In the work Rosenblatt demonstrates that Black fiction is a distinct body of writing, which has an identifiable, coherent literary tradition. He highlights recurring concerns and designs, which are independent of chronology in Black fiction written over the past eighty years. These concerns and designs are thematic, and they come form the central fact of the predominant white culture, where the Black characters in the novel are situated irrespective of whether they attempt to conform to that culture or they rebel against it. Rosenblatt's work does leave certain aesthetic questions open. His thematic analysis allows considerable objectivity; he even clearly states that he does not intend to judge the merit of the various works yet his reluctance seems misplaced, especially since an attempt to appraise might have led to interesting results. For example, certain novels have an appearance of structural diffusion. Is this a defeat, or are the authors working out of, or attempting to forge, a different kind of aesthetic? Apart from this, the style of certain Black novels, like Jean Toomer's Cane, verges on expressionism or surrealism; does this technique provide a counterpoint to the prevalent theme that portrays the fate against which Black heroes are pitted, a theme usually conveyed by more naturalistic modes of expressions? Irrespective of such omissions, what Rosenblatt talks about in his work makes for an astute and worthwhile study. His book very effectively surveys a variety of novels, highlighting certain fascinating and little-known works like James Weldon Johnson's Autobiography of an Ex-Coloured Man. Black Fiction is tightly constructed, and levelheaded and penetrating criticism is exemplified in its forthright and lucid style. 1. The author of the passage raises and objection to criticism of Black fiction like that by Addison Gayle as it: A. Highlights only the purely literary aspects of such works B. Misconceive the ideological content of such fiction C. Miscalculate the notions of Black identity presented in such fiction D. Replaces political for literary criteria in evaluating such fiction E. Disregards the reciprocation between Black history and Black identity exhibited in such fiction. Ans : D 2. The primary concern of the author in the above passage is: A. Reviewing the validity of a work of criticism

B. C. D. E.

Comparing various critical approaches to a subject Talking of the limitations of a particular kind of criticism Recapitulation of the major points in a work of criticism Illustrating the theoretical background of a certain kind of criticism.

Ans : A 3. The author is of the opinion that Black Fiction would have been improved had Rosenblatt: A. Undertaken a more careful evaluation of the ideological and historical aspects of Black Fiction B. Been more objective in his approach to novels and stories by Black authors C. Attempted a more detailed exploration of the recurring themes in Black fiction throughout its history D. Established a basis for placing Black fiction within its own unique literary tradition E. Calculated the relative literary merit of the novels he analyzed thematically. Ans : E 4. Rosenblatt's discussion of Black Fiction is : A. Pedantic and contentious B. Critical but admiring C. Ironic and deprecating D. Argumentative but unfocused E. Stilted and insincere. Ans : B 5. According to the given passage the author would be LEAST likely to approve of which among the following? A. Analyzing the influence of political events on the personal ideology of Black writers B. Attempting a critical study, which applies sociopolitical criteria to the autobiographies of Black authors C. A literary study of Black poetry that appraises the merits of poems according to the political acceptability of their themes D. Studying the growth of a distinct Black literary tradition within the context of Black history E. Undertaking a literary study, which attempts to isolate aesthetic qualities unique to Black fiction. Ans : C 6. From the following options, which does the author not make use of while discussing Black Fiction? A. Rhetorical questions

B. C. D. E.

Specific examples Comparison and contrast Definition of terms Personal opinion.

Ans : D 7. The author makes a reference to James Weldon Johnson's Autobiography of an Ex-colored Man most probably to: A. Highlight the affinities between Rosenblatt's method of thematic analysis and earlier criticism B. Elucidate regarding the point made regarding expressionistic style earlier in the passage C. Qualify the assessment of Rosenblatt's book made in the first paragraph of the passage D. Demonstrate the affinities among the various Black novels talked of by Rosenblatt's literary analysis E. Present a specific example of one of the accomplishments of Rosenblatt's work. Ans : E

5. Some modern anthropologists hold that biological evolution has shaped not only human morphology but also human behavior. The role those anthropologists ascribe to evolution is not of dictating the details of human behavior but one of imposing constraints - ways of feeling, thinking, and acting that ''come naturally'' in archetypal situations in any culture. Our ''frailties'' - emotions and motivs such as rage, fear, greed, gluttony, joy,lust, love-may be a very mixed assortment quality: we are, as we say, ''in the grip'' of them. And thus they give us oursense of constraints. Unhappily, some of those frailties our need for ever-increasing security among them are presently maladaptive. Yet beneath the overlay of cultural detail, they, too, are said to be biological in direction, and therefore as natural to us as are our appendixes. We would need to comprehend throughly their adaptive origins in order to understand how badly they guide us now. And we might then begin to resist their pressure. 1. The author implies that control to any extent over the ''frailties'' that constrain our behavior is though to presuppose A. That those frailties and adaptive are recognized as currently beneficial and adaptive B. That there is little or no overlay of cultural detail that masks their true nature. C. That there are cultures in which those frailties do not ''come naturally'' and from which such control can be learned

D. A full understanding of why those frailties evolved and of how they function now E. A thorough grasp of the principle that cultural detail in human behavior can differ arbitrarily from society to society. Ans : D 2. It can be inferred that in his discussion of maladaptive frailties the author assumes that A. Evolution does not favor the emergence of adaptive characteristics over the emergence of maladaptive ones B. Any structure or behavior not positively adaptive is regarded as transitory in evolutionary theory C. Maladaptive characteristics, once fixed, make the emergence of other maladaptive characteristics more likely D. The designation of a characteristic as being maladaptive must always remain highly tentative E. Changes in the total human environment can outpace evolutionary change. Ans : E 3. The primary purpose of the passage is to present A. A position on the foundations of human behavior and on what those foundations imply B. A theory outlining the parallel development of human morphology and of human behavior C. A diagnostic test for separating biologically determined behavior patters from culture - specific detail D. An overview of those human emotions and motive's that impose constraints on human behaviour E. A practical method for resting the pressures of biologically determined drives. Ans : A 4. Which of the following most probably provides an appropriate analogy from human morphology for the ''details'' versus ''constraints'' distinction made in the passage in relation to human behaviour? A. The ability of most people to see all the colors of the visible spectrum as against most peoples inability to name any but the primary colors B. The ability of even the least fortunate people to show compassion as against people's inability to mask their feelings completely C. The ability of some people to dive to great depths as against most people's inability to swim long distance D. The psychological profile of those people who are able to delay gratification as against people's inability to control their lives completely

E. The greater lung capacity of mountain peoples that helps them live in oxygen-poor air as against people's inability to fly without special apparatus. Ans : E

6. The existence of mammals on the earth can be traced back to at least the Triassic time. The rate of development was retarded, till evolutional change suddenly accelerated in the oldest Paleocene. This resulted in an increase in average size, larger mental capacity, and special adaptations for different modes of life, during the Eocene time. Further improvement was seen during the Oligocene Epoch, with the appearance of some new lines and extinction of others. The Miocene and Pliocene times are especially significant as they mark the culmination of various groups and a continued approach toward modern characters. It is in the Miocene time that the mammals reached their peak with reference to variety and size. The ability of the mammals to adapt to various modes of life finds a parallel in the reptiles of the Mesozoic time, and apart form their greater intelligence, the mammals apparently have not done much better than the corresponding reptilian forms. Undoubtedly the bat is a better flying animal than the pterosaur, but at the same time the dolphin and whale are hardly more fish like than the ichthyosaur. Quite a few of the swift-running mammals inhabiting the plains, like the horse and the antelope, must excel any of the dinosaurs. Although the tyrannosaur was a more weighty and robust carnivore than perhaps any carnivorous mammal, the lion and the tiger, by virtue of their superior brain are far more efficient and dangerous beasts of prey. It is significant to note that various species of mammals gradually adapted themselves to various kinds of lifestyles, some took to grazing on the plains and were able to run swiftly (horse, deer, bison), others started living in rivers and swamps (hippopotamus, beaver), inhabiting trees (sloth, monkey), burrowing underground (rodent, mole), feeding on flesh (tiger, wolf), swimming in the water (dolphin, whale, seal), and flying in the air (bat). Human beings on account of their superior brain have been able to harness mechanical methods to conquer the physical world and adapt to any set of conditions. Such adaptation to different conditions leads to a gradual change in form and structure. This is a biological characteristic of the youthful, plastic stage of a group. It is seen that early in its evolutional cycle animals possess the capacity for change, but as the animal progresses in its cycle becoming old and fixed, this capacity for change disappears. The generalized types of organisms retain longest the ability to make adjustments when required, and it is from them that new, fecund stocks take origincertainly not from any specialized end products. With reference to mammals, we see their birth, plastic spread in many directions, increased specialization, and in some cases, extinction; this is a characteristic of the evolution of life, which can be seen in the geologic record of life. 1. From the following, choose the most appropriate title for the above passage? A. From Dinosaur to Man

B. C. D. E.

Adaptation and Extinction The Superior Mammals The Geologic Life Span Man, the Vanquisher of the Physical World.

Ans : B 2. According to the passage the chronological order of the geologic periods is: A. Paleocene, Miocene, Triassic, Mesozoic B. Paleocene, Triassic, Mesozoic, Miocene C. Miocene, Paleocene, Triassic, Mesozoic D. Mesozoic, Oligocene, Paleocene, Miocene E. Mesozoic, Paleocene, Eocene, Miocene Ans : E 3. From the above passage, we can infer that, the pterosaur A. resembled the bat B. was a Mesozoic mammal C. was a flying reptile D. inhabited the seas E. evolved during the Miocene period Ans : C 4. As inferred from the passage, the largest number of mammals were found in which of the following periods? A. Triassic period B. Eocene period C. Oligocene epoch D. Pliocene period E. Miocene period Ans : E 5. Among the following statements, which statement, if true, would weaken the argument put forth in the first sentence of Paragraph 1? A. It has been found that the tryannosaur had a larger brain, than was previously known. B. Within the next thousand years, mammals will become extinct. C. Recently certain forms of flying ichthyosaurs have been discovered. D. It has now been proved, that the tiger is more powerful than the carnivorous reptiles. E. It is now possible to double human mental capacity, by the use of certain recently developed computers. Ans : A

6. It is clear from the passage, that the evidence used to discuss the life of past time periods A. was developed by Charles Darwin B. was unearthed by the author C. has been negated by more recent evidence D. was never truly established E. is based on fossilized remains Ans : E 7. As inferred from the passage, which of the following proverbial expressions is the author most likely to agree with? A. It's a cruel world. B. All the world's a stage. C. The more things change, the more they remain the same. D. Footprints in the sands of time. E. A short life, but a merry one. Ans : D

7. For a period of more than two centuries paleontologists have been intrigued by the fossilized remains of pterosaurs, the first flying vertebartes. The issues, which puzzle them, are how these heavy creatures, having a wingspan of about 8-12 meters managed the various problems associated with powered flight and whether these creatures were reptiles or birds. Perhaps the least controversial assertion about the pterosaurs is that they were reptiles. Their skulls, pelvises, and hind feet are reptilian. The anatomy of their wings suggests that they did not evolve into the class of birds. In pterosaurs a greatly elongated fourth finger of each forelimb supported a winglike membrane. The other fingers were short and reptilian, with sharp claws. In birds the second finger is the principal strut of the wing, which consists primarily of feathers. If the pterosaurs walked on all fours, the three short fingers may have been employed for grasping. When a pterosaurs walked or remained stationary, the fourth finger, and with it the wing, could only urn upward in an extended inverted V- shape along each side of the animal's body. In resemblance they were extremely similar to both birds and bats, with regard to their overall body structure and proportion. This is hardly surprising as the design of any flying vertebrate is subject to aerodynamic constraints. Both the pterosaurs and the birds have hollow bones, a feature that represents a savings in weight. There is a difference, which is that the bones of the birds are more massively reinforced by internal struts. Although scales typically cover reptiles, the pterosaurs probably had hairy coats. T.H. Huxley reasoned that flying vertebrates must have been warm-blooded because flying implies a high rate of metabolism, which in turn implies a high internal temperature. Huxley speculated that a coat of hair would insulate against loss of body heat and might streamline the body to reduce drag in flight. The recent discovery of a pterosaur

specimen covered in long, dense, and relatively thick hair like fossil material was the first clear evidence that his reasoning was correct. Some paleontologists are of the opinion that the pterosaurs jumped from s dropped from trees or perhaps rose into the light winds from the crests of waves in order to become airborne. Each theory has its associated difficulties. The first makes a wrong assumption that the pterosaurs hind feet resembled a bat's and could serve as hooks by which the animal could hang in preparation for flight. The second hypothesis seems unlikely because large pterosaurs could not have landed in trees without damaging their wings. The third calls for high aces to channel updrafts. The pterosaurs would have been unable to control their flight once airborne as the wind from which such waves arose would have been too strong. 1. As seen in the above passage scientists generally agree that: A. the pterosaurs could fly over large distances because of their large wingspan. B. a close evolutionary relationship can be seen between the pterosaurs and bats, when the structure of their skeletons is studied. C. the study of the fossilized remains of the pterosaurs reveals how they solved the problem associated with powered flight D. the pterosaurs were reptiles E. Pterosaurs walked on all fours. Ans : D 2. The view that, the pterosaurs rose into light winds from the crest of the waves to become airborne, is viewed by the author as A. revolutionary B. unlikely C. unassailable D. probable E. outdated. Ans : B 3. As inferred from the passage, the skeleton of a pterosaur is distinguishable from that of a bird by the A. length of its wingspan B. hollow spaces in its bones C. anatomic origin of its wing strut D. evidence of the hooklike projections on its hind feet E. location of the shoulder joint joining the wing to its body. Ans : C 4. From the viewpoint of T.H.Huxley, as given in the passage, which of the following statements is he most likely to agree with? A. An animal can master complex behaviors irrespective of the size of it's brain.

B. Environmental capabilities and physical capabilities often influence the appearance of an animal. C. Usually animals in a particular family group do not change their appearance dramatically over a period of time D. The origin of flight in vertebrates was an accidental development rather than the outcome of specialization or adaption E. The pterosaurs should be classified as birds, not reptiles. Ans : B 5. According to the passage which of the following is a characteristic of the pterosaurs? A. The pterosaurs were not able to fold their wings when not in use B. Like the bats, they hung upside down from branches C. They flew in order to capture prey D. They can be said to be an earlier stage in the evolution of the birds E. They lived principally in a forest like habitat. Ans : A 6. The organization of the last paragraph of the passage can best be described as: A. New data is introduced in order to support a traditional point of view B. Three explanations are put forth and each of them is disputed by means of specific information C. An outline of three hypotheses are given and evidence supporting each of them is given D. Description of three recent discoveries is presented, and their implications for future study are projected E. The material in the earlier paragraphs is summarized and certain conclusions are from it. Ans : B 7. According to the passage, some scientists believe that pterosaurs A. Lived near large bodies of water B. Had sharp teeth for tearing food C. Were attacked and eaten by larger reptiles D. Had longer tails than many birds E. Consumed twice their weight daily to maintain their body temperature. Ans : A

8. Certain scraps of evidence bear out those who hold a very high opinion of the average level of culture among the Athenians of the great age. Pericles's funeral speech is undoubtedly the most famous evidence from Athenian literature, that its level was indeed high. However, Pericles was a politician,

and it is possible that he was flattering his audience. We know that thousands of Athenians sat hour after hour in the theater listening to the plays of the great Greek dramatists. The Greek plays, particularly the tragedies, maintained an extremely high intellectual level throughout, with no letdowns, no concessions to the lowbrows or to the demands of ''realism'', like the gravediggers scene in Shakespeare's Hamlet. The music and dancing seen in these plays were also of an equally high level. The best modern parallel can be seen in the restrained, difficult opera of the 18th century. The comparison is no doubt dangerous, but can you imagine almost the entire population of an American city (in suitable installments, of course) sitting through performances of Mozart's Don Giovanni or Gluck's Orpheus? Perhaps the Athenian masses went to these plays because of a lack of other amusements. They could at least understand something of what went on, since the subjects were part of their folklore. Undoubtedly the theme of grand opera is not part of the folklore of the American people. 1. From the passage it is evident that the author seems to question the sincerity of A. politicians B. playwrights C. opera goers D. ''low brows'' E. gravediggers. Ans : A 2. According to the author the average American A. Enjoys Hamlet B. Loves folklore C. Is not able to understand grand opera D. Seeks a high cultural level E. Lacks entertainment. Ans : C 3. From the passage, we can say that the author's attitude toward Greek plays is one of A. Qualified approval B. Grudging admiration C. Studied indifference D. Partial hostility E. Great respect. Ans : E 4. The author makes a suggestion that Greek plays A. Were demanding on the actors B. Flattered their audiences C. Were focussed on a limited audience D. Were dominated by music and dancing E. Stimulated their audiences.

Ans : E

9. Everyone conforms to infancy, infancy conforms to nobody, so that one babe commonly makes four or five out of the adults who prattle and play to it. So God has armed youth and puberty and manhood no less with its own piquancy and charm, and made it enviable and gracious and its claims not to be put by, if it will stand by itself. Do not think the youth has no force, because he cannot speak to you and me. Hark! In the next room his voice is sufficiently clear and emphatic. It seems he knows how to speak to his contemporaries. Bashful or bold, then, he will know how to make us seniors very unnecessary. The healthy attitude of human nature can be seen in the nonchalance of boys who are sure of a dinner, and would disdain as much as a lord to do or say aught to conciliate one. A boy is in the parlor what the pit is in the playhouse; independent, irresponsible, looking out from his corner on such people and facts as pass by, he tries and sentences them on their merits, in the swift, summary way of boys, as good, bad, interesting, silly, eloquent, troublesome. He never cumbers himself regarding consequences, about interests and he gives an independent, genuine verdict. You should court him: he will not court you. But the man is, as it were, clapped into jail by his consciousness. As soon as he has once acted or spoken with eclat, he is a committed person, watched by the sympathy or the hatred of hundreds, whose affections must now enter into his account. There is no Lethe for this. Ah, that he could pass again into his neutrality. These are the voices, which we hear in solitude, but they grow faint and inaudible as we enter into the world. Everywhere society is conspiring against the manhood of every one of its members. Society is joint stock company, in which members agree, for the better securing of his bread to each shareholder, to surrender the liberty and culture of the eater. The virtue in most request is conformity. It is averse to selfreliance. What it loves is names and customs and not realities and creators. Whosoever is a man has to be a nonconformist. He who would gather immortal palms must not be hindered by the name of goodness, but must explore if it be goodness. Nothing is at last sacred but the integrity of your own mind. No law can be sacred to me but that of my nature. Good and bad are but names very readily transferable to that to this; the only right is what is after my constitution, the only right is what is after me constitution, the only wrong what is against it. A man is to carry himself in the presence of all opposition as if every thing were titular and ephemeral but he. I am ashamed to think how easily we capitulate to badges and names, to large societies and dead institutions. Every decent and well-spoken individual affects and sways me more than is right. I ought to go upright and vital, and speak the rude truth in all ways.

I shun father and mother and wife and brother, when my genius calls me. I would write on the lintels of the doorpost, whim. I hope it is somewhat better than whim at last, but we cannot spend the day in explanation. Except me not to show cause why I seek or why I exclude company. Then, again, do not tell me, as a good man did not today, of my obligation to put all poor men in good situations. Are they my poor? I tell thee, thou foolish philanthropist, that I grudge the dollar, the time, the cent, I give to such men as do not belong to me and to whom I do not belong. There is a class of person to whom by all spiritual affinity I am bought and sold; for them I will go to prison, if need be; but your miscellaneous popular charities; the education at collage of fools; the building of meeting house to the vain end to which many now stand; alms to sots; and the thousandfold Relief Societies; - though I confess with shame I sometimes succumb and give the dollar, it is a wicked dollar which by and by I shall have the manhood to withhold. If you refuse to conform, you can experience the displeasure of the world. Hence, a man should know how to estimate a sour face. The by standers look askance on him in the public street or in the friend's parlor. In case this aversion originates from contempt and resistance similar to his own, it might result in a sad countenance; but the sour faces of the multitude, like their sweet faces, have no deep cause, but are caused by reasons as diverse as the direction of the wind and what he reads in the newspapers. Yet is the discontent of the multitude more formidable than that of the senate and the collage. Another factor, which frightens us from self trust in our consistency; a reverence for our past act or word, because the eyes of others have no other data for computing our orbit than our past acts, and we are loath to disappoint them. But why should you keep your head over your shoulder? Why drag about this corpse of your memory, lest you contradict somewhat you have stated in this or that public place? Suppose you should contradict yourself; what then? This is a rather silly consistency in our minds, which is adored by little statesmen and philosophers and divines. Uniformly a great soul has almost nothing to do, he could just occupy himself with his shadow on the wall. Speak what you think now in hard words; and to-morrow speak what tomorrow thinks in hard words again, though it contradict everything you said to-day. ''Ah, so you shall be sure to be misunderstood.'' - Is it so bad, then, to be misunderstood? Pythagoras was misunderstood, and Socrates, and Jesus, and Luther, and Copernicus, and Galileo, and Newton, and every pure and wise spirit that ever took flesh. What can be considered to be truly great is to be misunderstood. 1. Which of the following statements would best describe the main theme of the above passage? A. "A foolish consistency is the hobgoblin of little mind." B. "Eternal youth means eternal independence." C. "Whoso would be a man must be a nonconformist." D. "Colleges are designed to educate fools." E. "Infancy conforms to nobody."

Ans : C 2. When is the period during which we are most nonconformist? A. infancy B. puberty C. youth D. manhood E. old age Ans : A 3. In his statement ''What can be considered to be truly great is to be misunderstood'' the author means: A. One should refrain from saying, what one exactly means B. Being misunderstood, equals being great C. All great man have always been misunderstood D. Even though a person might be considered inconsistent, he shouldn't hesitate to change his mind if he feels the need to. E. It is seldom, that nice people succeed Ans : D 4. As inferred from the passage, the refusal of young people to cater to accept public opinion is: A. A feature of the rebelliousness of youth B. A healthy attitude of human nature C. A manifestation of deep- seated immaturity D. Simply bad manners E. Part of growing up Ans : B 5. "Society is a joint-stock company etc." is one way which the author shows A. The anti-culture attitude of the public B. Society is highly organized and structured C. The self-rejection of society D. The lack of room for solitude in our world E. The public's interest in the stock market Ans : C 6. " I would write on the lintels of the doorpost, whim." What does the author mean by this statement: A. That one should renounce his immediate family B. That signposts have an important educational function in our society C. That an impulsive action may have a subsequent rational explanation D. That one must never be held responsible for what one says and does E. That everyone should do foolish things occasionally

Ans : C 7. Which of the following statements best summarizes the spirit and sense of the above passage? A. "Nothing is at last sacred but the integrity of your own mind." B. "With consistency, a great soul; has simply nothing to do." C. "Do not think the youth has no force, because cannot speak to you and me." D. "The virtue in most request is conformity." E. "A man must know how to estimate a sour force." Ans : A

10. Furthermore, insofar as any conclusion about its author can be drawn from five or six plays attributed to him, the Wakefield Master is without exception considered to be a man of sharp contemporary observation. He was, probably clerically educated, as indicated by his Latin and music, his Biblical and patristic lore. Even today he is remembered for his his quick sympathy for the oppressed and forgotten man, his sharp eye for character, a ready ear for colloquial, vernacular turns of speech and a humor alternately rude and boisterous, coarse and happy. Therefore in spite of his conscious artistry as can be seen in his feeling for intricate metrical and stanza forms, he is regarded as a kind of medieval Steinbeck, indignantly angry at, uncompromisingly and even brutally realistic in presenting the plight of the agricultural poor. It is now fairly accepted to regard the play as a kind of ultimate point in the secularization of the medieval drama. Therefore more stress has been laid on it as depicting realistically humble manners and pastoral life in the bleak of the west riding of Yorkshire on a typically cold night of December 24th. After what are often regarded as almost ''documentaries'' given in the three successive monologues of the three shepherds, critics go on to affirm that the realism is then intensified into a burlesque mock-treatment of the Nativity. Finally as a sort of epilogue or after-thought in deference to the Biblical origins of the materials, the play slides back into an atavistic mood of early innocent reverence. In actuality, the final scene is the culminating scene and also the raison detre of the introductory ''realism.'' Superficially the present play supports the conventional view of its mood of secular realism. At the same time, the ''realism'' of the Wakefield Master is of a paradoxical turn. His wide knowledge of people, as well as books indicates no cloistered contemplative but one in close relation to his times. Still, that life was after all a predominantly religious one, a time which never neglected the belief that man was a rebellious and sinful creature in need of redemption . So

deeply (one can hardly say ''naively'' of so sophisticated a writer) and implicitly religious is the Master that he is less able (or less willing) to present actual history realistically than is the author of the Brome Abraham and Isaac. His historical sense is even less realistic than that of Chaucer who just a few years before had done for his own time ''costume romances,'' such as The Knight's Tele, Troilus and Cressida, etc. Furthermore, used highly romantic materials, which could excuse his taking liberties with history. 1. Of the following statements, which is not true of Wakefield Master? A. He and Chaucer were contemporaries. B. Wakefield Master is remembered as having written five or six realistic plays. C. His plays realistically portray the plight of the country folk of his day D. His writing was similar to that of John Steinbeck. E. He was an accomplished artist. Ans : D 2. The word 'patristic' in the first paragraph is used to mean: A. patriotic B. superstitious C. folk D. relating to the Christian Fathers E. realistic Ans : D 3. The statement about the ''secularization of the medieval drama'' (opening sentence of the second paragraph) refers to the A. Introduction of religious themes in the early days B. Presentation of erudite material C. Use of contemporary materials D. Return to early innocent reverence at the end of the play E. Introduction of mundane matters in religious plays Ans : E 4. From the following what would the writer be expected to do in the subsequent paragraphs: A. Make a justification for his comparison with Steinbeck B. Put forth a view point, which would take up the thought of the second paragraph C. Point out the anachronisms in the play D. Discuss the works of Chaucer E. Talk about the lack of realism in the works of the Wakefield Master.

Ans : B

11. The establishment of the third Reich influenced events in American history by starting a chain of events which culminated in war between Germany and the United States. The complete destruction of democracy, the persecution of laws, the war on religion, the cruelty and barrbarism of the Nazis and especially, the plans of Germany and her allies, Italy and Japan, for world conquest caused great indignation in this country and brought on fear of another world war. While speaking out against Hitler's atrocities, the American profile generally favored isolationist policies, and neutrality. The neutrality acts of 1935 and 1936 prohibited trade with any belligerents or loans to them. In 1937 the president was empowered to declare an arms embargo in wars between nations at his discretion American opinion began to change somewhat after President Roosevelt's quarantine the aggvessor speech at Chicago (1937) in which he severely criticized Hitler's policies. Germany's seizure of Austria and Munich pact for the partition of Czechoslovakia (1938) also around the American people. The conquest of Czechoslovakia in March 1939 was another rude awakening to the menace of the third Reich. In August, 1939, came the shock of the Nazi Soviet pact and in September the attack on Poland and the outbreak of European war. The United States attempt to maintain neutrality in spite of sympathy for the democracies arranged against the Third Reich. The Neutrality act of 1939 repeated the arms embargo and permitted 'cash' and 'carry' exports of arms to belligerent nations. A strong national defense program was begun. A draft act was passed (1940) to strengthen the military services. A Lend - Lease Act (1940) authorized the president to sell, exchange or lend materials to any county deemed necessary by him for the defense of the United States. Help was given to Britain territory in the western Hemisphere. In August 1941, President Roosevelt and prime minister Churchill met and issued the Atlantic Charter which proclaimed the kind of a world which should be established after the war. In December 1941, Japan launched the unprovoked attack on the United States at Pearl harbor, immediately thereafter Germany declared war on the united states. 1. USA entered the war against Germany A. because Pearl Harbor was attacked B. after peaceful efforts had failed C. because Germany declare war against it D. because Japan was an ally of Germany E. after Germany had signed the Nazi - Soviet pact Ans : C

2. The Neutrality Act of 1939 favored Great Britain because A. the British had command of the sea B. the law permitted U.S.A. to trade only with the allies. C. it antagonized Japan D. it led to the Land - Lease Act E. it agreed with the British on the principle of the Atlantic Charter Ans : A 3. An event that did not occur in 1939 was the A. invasion of Poland B. invasion of Czechoslovakia C. passing of the Neutrality Act D. passing of the Land - Lease act E. outbreak of the war in Europe Ans : D 4. One item occurring 1937 that the author does not mention in the list of actions that alienated the American Public was A. The persecution of religious groups B. Nazi barbarism C. The pacts with Italy D. German plans for conquest of the world E. The burning of the Reich tag. Ans : E 5. The Land - Lease Act has designed to A. Strengthen USA's national defense B. Provide battle shit to the Allies C. Help the British D. the Atlantic Charter E. Avenge Pearl Harbor Ans : A 6. The Neutrality Act of 1939 A. restated America's isolationist policies B. proclaimed American neutrality C. permitted the selling of arms to belligerent nation D. was cause of USA's entrances in to WORLD WAR II E. started USA's national defense programs Ans : C 7. During the years 1933-36, American policy may be described as having been

1. 2. 3. 4. 5.

watchful isolationist pacific incorrect discretionary

Ans : B

12. There was in increase of about 10 % in the investment in the public sector, like electricity, irrigation quarrying, public services and transport; even though the emphasis leaned towards transport and away from the other sectors mentioned. A 16-17% growth in investment, including a 30% increase in investment in business premises has been recorded in trade and services. Although there continued to be a decline in the share of agriculture in total gross investment in the economy, investment grew by 9% in absolute terms, largely spurred on by a 23% expansion of investment in agriculture equipment. Housing construction had 12% more invested in it in 1964, not so much owing to increase demand, as to fears of impending new taxes and limitation of building. There was a rise of close to 11% in the total consumption in real terms during 1964 and per capita personal consumption by under 7%, as in 1963. The undesirable trend towards a rapid rise in consumption, evident in previous years, remains unaltered. Since at current prices consumption rose by 16% and disposable income by 13%, there was evidently a fall in the rate of saving in the private sector of the economy. Once again a swift advance in the standard of living was indicated in consumption patterns. Though fruit consumption increased, expenditure on food, especially bread and staple items, declined significantly. There was a continuing increase in the outlay on furniture and household equipment, health, education and recreation. The greatest proof of altered living standards was the rapid expansion of expenditure on transport (including private cars) and personal services of all kinds, which occurred during 1964. The changing composition if purchased durable goods demonstrated the progressive affluence of large sectors of the public. On the one hand increased purchase of automobiles and television sets were registered, a point of saturation was rapidly being approached for items like the first household radio, gas cookers, and electric refrigerators. 1. It is possible to to conclude from this passage, that the people of the country were A. spending more money than they earn B. investing and consuming at an accelerated pace C. saving more money than previously D. spending their money wisely E. lacking in necessities Ans : B

2. According to the author the trend towards a rapid rise in consumption is "undesirable" as: A. there was an increase in the expenditure on frills and luxuries B. the people were affluent C. there was a rise in the standard of living D. people were eating less E. people were saving less Ans : E 3. It is possible to conclude that the United States is not the discussed country as: A. there was a decline in the expenditures for food B. From the statement that the saturation point was rapidly being approached for first household radios C. there is no mention of military expenditures D. the people were affluent E. the people were not saving their money Ans : B 4. The area, which saw the greatest expenditure of investment funds was A. The public sector B. Business premises C. Housing construction D. Agricultural equipment E. A field which cannot be determined Ans : E

13. Visual recognition involves storing and retrieving memories. Neural activity, triggered by the eye, forms an image in the brains memory system that constitutes an internal representation of the viewed object. When an object is encountered again, it is matched with its internal representation and thereby recognized. Controversy surrounds the question of whether recognition is a parallel, one-step process or a serial, step-by-step one. Psychologists of the Gestalt school maintain that object are recognized as wholes in a parallel procedure : , the internal representation is matched with the retinal image in a single operation. Other psychologists have proposed that internal representation features are matched serially with an object's features. Although some experiments show that, as an object become familiar, its internal representation becomes more familiar, its internal representation becomes more holistic and the recognition process correspondingly more parallel, the weight of evidence seems to support the serial hypothesis, at least for objects that are not notably simple and familiar.

1. It can be inferred from the passage that the matching process in visual recognition is A. Not a natural activity. B. Not possible when an object is viewed for the very first time. C. Not possible if a feature of a familiar object is changed in same way. D. Only possible when a retinal image is received in the brain as a unitary whole. E. Now fully understood as a combination of the serial and parallel process. Ans : A 2. In terms of its tone and form, the passage can best be characterized as A. A biased exposition B. A speculative study C. A dispassionate presentation D. An indignant denial E. A dogmatic explanation. Ans : C 3. The author is primarily concerned with A. Explaining how the brain receives images B. Synthesizing hypotheses of visual recognition C. Examining the evidence supporting the serial recognition hypothesis D. Discussing visual recognition and some hypotheses proposed to explain it. E. Reporting on recent experiments dealing with memory systems and their relationship to neural activity. Ans : B 4. According to the passage, Gestalt psychologists make which of the following suppositions about visual recognition? I A retinal image is in exactly the same form as its internal representation II An object is recognized as a whole without any need for analysis into component parts. III The matching of an object with its internal representation occurs in only one step A. II only B. III only C. I and III only D. II and III only E. I, II and III

Ans : D

14. According to Albert Einstein the non mathematician, is seized by a mysterious shuddering when he hears of 'four-dimensional' things, he is seized by a feeling, which is very similar to the thoughts awakened by the occult. And at the same time the statement that the world in which we live is a fourdimensional space - time continuum is quite a common place statement. This might lead to an argument regarding the use of the term ''commonplace'' by Einstein. Yet the difficulty lies more in the wording than the ideas. Einstein's concept of the universe as a four-dimensional space-time continuum becomes plain and clear, when what he means by ''continuum'' becomes clear. A continuum is something that is continuous, A ruler, for example, is a onedimensional space continuum. Most rulers are divided into inches and fractions, scaled down to one-sixteenth of an inch. Will it be possible to conceive a ruler, which is calibrated to a millionth or billionth of an inch. In theory there is no reason why the steps from point to point should not be even smaller. What distinguishes a continuum is the fact that the space between any two points can be sub-divided into an infinite number of smaller divisions. A railroad track is a one-dimensional space continuum and on it the engineer of a train can describe his position at any time by citing a single co-ordinate point - i.e., a station or a milestone. A sea captain, however, has to worry about two dimensions. The surface of the sea is a two-dimensional continuum and the co-ordinate points by which sailor fixes his positions in his two dimensional continuum are latitude and longitude. An airplane pilot guides his plane through a three - dimensional continuum, hence he has to consider not only latitude and longitude, but also his height above the ground. The continuum of an airplane pilot constitutes space as we perceive it. In other words, the space of our world is a three-dimensional continuum. Just indicating its position in space is not enough while describing any physical event, which involves motion. How position changes in time also needs to be mentioned. Thus to give an accurate picture of the operation of a New York - Chicago express, one must mention not only that it goes from New - York to Albany to Syracuse to Cleveland to Toledo to Chicago, but also the times at which it touches each of those points. This can be done either by means of a timetable or a visual chart. If the miles between New York and Chicago are plotted horizontally on a piece of ruled paper and the hours and minutes are plotted vertically, then a diagonal line properly drawn across the page illustrates the progress of the train in two - dimensional space - time continuum. This type of graphic representation is familiar to most newspaper

readers; a stock market chart, for example, pictures financial events in a two dimensional dollar - time continuum. Similarly for the best picturization of the flight of an airplane from New York to Los Angeles a four - dimensional space time continuum is essential. The latitude, longitude and altitude will only make sense to the traffic manager of the airline if the time co - ordinate is also mentioned. Therefore time is the fourth dimension. If a flight has to be looked at, perceived as a whole, it wouldn't work if it is broken down into a series of disconnected take - offs, climbs, glides, and landing, it needs to be looked at and perceived as a continuous four - dimensional space - time continuum curve. 1. In order to explain a difficult topic, the author use A. Simply phrased definition's B. An incessant metaphor C. A plain writing style D. Familiar images E. A quotation from Einstein Ans : D 2. The significant feature of a continuum, according to the passage, revolves around A. The divisibility of the interval between any two points. B. An ordinary ruler's caliber for marking C. Its unending curve D. Its lucid from providing comprehensibility to the non scientists as well E. Its variety of co - ordinates. Ans : A 3. The purpose of this passage is to highlight the point that A. Plots and sea captains have something in common B. Stock market charts may be helpful to physicists C. The fourth dimension is time. D. Non - mathematician's are often afraid of the commonplace E. There is a marked quality to distance Ans : C 4. According to the passage, an airlines traffic manager depends upon all of the following EXCEPT A. latitude B. altitude C. the time co - ordinate D. longitude E. the continuous curve in co four Ans : E

5. The underlying tone of this selection is A. persuasive B. deferential C. candid D. instructive E. gently condescending Ans : D 6. According to the author if on wishes portray a physical event in which motion plays a role - one has to A. Make use of a time-table B. Indicate how position changes in time C. Be conversant with the scientist's theories D. Describe it graphically E. Be aware of altitude, latitude and longitude Ans : B 7. The sea-captain's example has been cited in order to A. Help understand a two - dimensional continuum B. Set up a logical progression C. Simplify what ever is too elaborate D. Mitigate the gap between the engineer and pilot E. To sustain out interest in the reading of the passage. Ans : A

15. From the 197 million square miles, which make up the surface of the globe, 71 per cent is covered by the interconnecting bodies of marine water; the Pacific Ocean alone covers half the Earth and averages near 14,000 feet in depth. The portions which rise above sea level are the continents-Eurasia, Africa; North America, South America, Australia, and Antarctica. The submerged borders of the continental masses are the continental shelves, beyond which lie the deep-sea basins. The ocean are deepest not in the center but in some elongated furrows, or long narrow troughs, called deeps. These profound troughs have a peripheral arrangement, notably around the borders of the pacific and Indian oceans. The position of the deeps, like the highest mountains, are of recent origin, since otherwise they would have been filled with waste from the lands. This is further strengthened by the observation that the deeps are quite often, where world-shaking earthquakes occur. To cite an example, the "tidal wave" that in April, 1946, caused widespread destruction along Pacific coasts resulted from a strong earthquake on the floor of the Aleutian Deep. The topography of the ocean floors is none too well known, since in great areas the available soundings are hundreds or even thousands of miles apart. However, the floor

of the Atlantic is becoming fairly well known as a result of special surveys since 1920. A broad, well-defined ridge-the Mid-Atlantic ridge-runs north and south between Africa and the two Americas and numerous other major irregularities diversify the Atlantic floor. Closely spaced soundings show that many parts of the oceanic floors are as rugged as mountainous regions of the continents. Use of the recently perfected method of submarine topography. During world war II great strides were made in mapping submarine surfaces, particularly in many parts of the vast Pacific basin. Most of the continents stand on an average of 2870 feet above sea level. North America averages 2300 feet; Europe averages only 1150 feet; and Asia, the highest of the larger continental subdivisions, averages 3200 feet. Mount Everest, which is the highest point in the globe, is 29,000 feet above the sea; and as the greatest known depth in the sea is over 35,000 feet, the maximum relief (that is, the difference in altitude between the lowest and highest points) exceeds 64,000 feet, or exceeds 12 miles. The continental masses and the deep-sea basins are relief features of the first order; the deeps, ridges, and volcanic cones that diversify the sea floor, as well as the plains, plateaus, and mountains of the continents, are relief features of the second order. The lands are unendingly subject to a complex of activities summarized in the term erosion, which first sculptures them in great detail and then tends to reduce them ultimately to sea level. The modeling of the landscape by weather, running water, and other agents is apparent to the keenly observant eye and causes thinking people to speculate on what must be the final result of the ceaseless wearing down of the lands. Much before there was any recognizable science as geology, Shakespeare wrote "the revolution of the times makes mountains level." 1. The peripheral furrows or deeps are found A. only in the pacific and Indian oceans B. near earthquakes C. near the shore D. in the center of the ocean E. to be 14,000 feet in depth in the pacific. Ans : C 2. The largest ocean is the A. Atlantic B. pacific C. Aleutian deep D. arctic E. Indian. Ans : B 3. We may conclude from this passage that earth quakes A. Occur more frequently in newly formed land or sea formations B. Are caused by the weight of the water C. Cause erosion D. Occur in the deeps

E. Will ultimately "make mountains level". Ans : A 4. The highest mountains are A. oldest B. in excess of 12 miles C. near the deeps D. relief features of the first order E. of recent origin. Ans : E 5. The science of geology was started A. By the Greeks B. During world war II C. April 1946 D. After 1600 E. In 1920 Ans : D 6. The highest point on North America is A. 2870 feet above sea level B. not mentioned in the passage C. higher than the highest point in Europe D. 2300 feet above sea level E. in Mexico. Ans : B 7. The deeps are subject to change caused by A. erosion B. soundings C. earthquakes D. waste E. weathering Ans : C 8. The continental masses A. Rise above sea level B. Consist of six continents C. Are relief features of the second order D. Are partially submerged E. Comprise 29 per cent of the earth's surface. Ans : D

16. A clear answer to whether the languages of the ancient American peoples were made use of for expressing abstract universal concepts can be sought in the case of Nahuatl, which like Greek and German, is a language that allows the formation of extensive compounds. By combining radicals or semantic elements, single compound words can express complex conceptual relations, often of an abstract universal character. The tlamatinime ("those who know") were able to use this rich stock of abstract terms to express the nuances of their thought. They also availed themselves of other forms of expression with metaphorical meaning, some probably original, some derived from Toltec coinages. Of these forms the most characteristic in Nahuatl is the juxtaposition of two words that, because they are synonyms, associated terms, or even contraries, complement each other to evoke one single idea. The juxtaposed terms, used as metaphor, suggest specific or essential traits of the being they refer to, introducing a mode of poetry as an almost habitual form of expression. 1. According to the passage, some abstract universal ideas can be expressed in Nahuatl by A. Putting various meaningful elements together in one word B. Taking away from a word any reference to particular instances C. Turning each word of a phrase into a poetic metaphor D. Giving a word a new and opposite meaning E. Removing a word from its associations with other words. Ans : A 2. It can be inferred solely from the information in the passage that A. Metaphors are always used in Nahuatl to express abstract conceptual relationships B. There are many languages that, like Greek or German, allow extensive compounding C. The abstract terms of the Nahuatl language are habitually used in poetry D. Some record or evidence of the though of the tlamatinime exists E. All abstract universal ideas are ideas of complex relations. Ans : D 3. A main purpose of the passage is to A. Argue against a theory of poetic expression by citing evidence about the Nahuatl B. Delineate the function of the tlamatinime in Nahuatl society C. Explore the rich metaphorical heritage the Nahuatl received from the toltecs

D. Describe some conceptual and aesthetic resources of the Nahuatl language E. Explain the abstract philosophy of the Nahuatl thinkers.

17. Few areas of neuron behavioral research seemed more promising is the early sixties than that investigating the relationship between protein synthesis and learning. The conceptual framework for the research was derived directly from molecular biology, which had shown that genetic information is stored in nucleic acids and expressed in proteins why not acquired information as well. The first step towards establishing a connection between protein synthesis and learning seemed to be to block memory (cause adhesion) by interrupting the production of proteins. We were fortunate in finding a non lethal dosage of puromycin that could, it first appealed, thoroughly inhibit brain protein synthesis as well as reliability produce amnesia. Before the actual connection between protein synthesis and learning could be established however we began to have douche about whether inhibition of protein synthesis was in fact the method by which puromycin produced amnesia. First, ocher drugs, glutavimides themselves potent protein synthesis inhibitors either failed to cause amnesia in some situations where it could easily be induced by puromycin or produced an amnesia with a different time course from that of puromycin. Second, puromycin was found to inhabit protein synthesis by breaking certain amino acid chaim, and the resulting fragments were suspected of being the actual cause of amnesia is some eases. Third, puromycin was reported to cause abnormalities in the train, including seizures. Thus, not only were decreased protein synthesis and amnesia dissociated, but alternative mechanism for the amnestic action of puromycin were readily suggested. So, puromycin turned out to be a disappointment. It came to be regarded as a poor agent for amnesia studies, although, of course, it was poor only in the context of our original paradigm of protein synthesis inhibition. In our frustration, our initial response was simply to change dregs rather than our conceptual orientation. After many such disappointments, however, it now appears unlikely, that we will make a firm connection between protein synthesis and learning merely by pursuing the approaches of the past our experience with drugs has shown that all the amnestic agents, often interfere with memory in ways that seem unrelated to their inhibition of protein synthesis. More importantly, the notion that the interruption or intensification of protein production in the train can be related in cause and affect fashion to learning non seems simplistic and unproductive. Remove the battery from a car and the car will not go Drive the car a long distance at high speed and the battery will become more highly charged. Neither of these facts proves that the battery power the car, only knowledge of the overall automotive system will reveal it mechanism of locomotion and the role of the battery with in the system.

1. The primary purpose a the passage is to show that extensive experimentation has A. Mot supported the hypothesis that learning is directly dependent on protein synthesis B. Cast doubt on the value of puromycin in the newer behavioral study of learning C. Revealed the importance of amnesia in the neuron behavioral study of learning D. Demonstrated the importance of amino acid fragmentation in the induction of amnesia. E. Not yet demonstrated the applicability of molecular biology to behavioral research. Ans : A 2. According to the passage, neuron behaviorists initially based their belief that protein synthesis was related to learning on which of the following? A. Specific research into learning on which of the following B. Traditional theories about learning C. Historic experiments on the effects puromycin D. Previous discoveries in molecular biology E. Now technique in protein synthesis. Ans : D 3. This passage was most likely excepted from A. A book review in a leading journal devoted to genetic research. B. A diary kept by a practicing neuron behavioral research C. An article summarizing a series of scientific investigations in neuron behavioral research. D. A news paper article on recent advances in the biochemistry of learning E. A technical article on experimental techniques in the field of molecular biology. Ans : C 4. It can be inferred from the passage that after puromycin was perceived to be a disappointment, researches did which of the following? A. They continued to experiment with puromycin until a neuron anatomical framework was developed. B. They continued to experiment with puromycin, but also tried other protein synthesis inhibitors C. They ceased to experiment with puromycin and shifted to other promising protein synthesis inhibitors.

D. They ceased to experiment with puromycin and reexamined through experiments the relationship between genetic information and acquired information. E. They continued to experiment with puromycin, but applied their results to other facts of memory research. Ans : C 5. In the example of the car (lines 62-70) the battery is meant to represent which of the following elements in the neuron behavioral research program? A. glutarimides B. acquired information C. puromycin D. amnesia E. protein synthesis Ans : E 6. The passage all of the following as effects of puromycin except A. Fragmentation of amino-acid chaim B. Inhibition of protein synthesis C. Brain seizures D. Memory loss E. Destruction of genetic information Ans : E 7. Which of the following statements would be most likely to come after the last sentences of the passage? A. It is important in the future, therefore for behavioral biochemist to focus on the several components of the total learning system. B. The ambivalent status of current research, however should not deter neuron behaviorists from exploring the deeper connection between protein production and learning. C. The failures of the past, however must not impede further research into the amnestic of protein-synthesis inhibitors. D. It is important in the future, therefore, for behavioral biochemist to emphasize more strongly place of their specific findings within the overall protein synthesis model of learning. E. It is a legacy of this research, therefore, that molecular biology's genetic models have led to disagreement among neuron behaviorists. Ans : A

18. In any country, the wages commanded by the laborers who have comparable skills but who work in various industries are determined by the productivity of the least productive unit of labour, i.e. the unit of labour which works in the industry which has catatest economic disadvantages. We will represent the various opportunities of employment in a country like united states by Symbols. A standing for a group of industries in which we have exceptional economic advantage over foreign countries; B for a group in which our advantages are less; E , one in which they are still less; D, the group of industries in which they are the least of all. When our population is so small that all our labour can be engaged in the group represented by A, productivity of labour and (therefore wages) will be at their maximum. when our population increases so that some of the labour will have to work in group B, the wages of all labour must decline to the level of productivity in that group. But no employer, without government aid, will yet be able to afford to hire labour to exploit the opportunities, represented by E and D, unless there is a further increase in population. But suppose that the political party in power holds the belief that we should produce everything that we consume, that the opportunities represented by E and D should also be exploited. The commodities, that the industries composing C and D will produce have been hitherto obtained from abroad in exchange for commodities produce by A and B. The government now renders this difficult by imposing high duties upon the former class of commodities. This means that workers in A and B must pay higher prices for what they buy, but do not receive higher prices for what they sell. After the duty has gone into effect and the prices of commodities that can be produced by C and D have risch sufficiently enterprises will be able to hire labour at the wages prevailing in A and B and establish industries in C and D. So far as the remaining labours in A and B buy the products of C and D ,the difference between the price which they pay for these product and the price they would pay it they were permitted to import those products duty-free is a tax paid not to the government, but to the producers in C and D, to enable the later to remain in business. It is on uncompensated deduction from the natural earnings of the labourers in A and B. nor are the workers in C and D paid as much, estimated in purchasing power as they would have received if they had been allowed to remain in A and B under the earlier conditions. 1. The authors main point is that A. The government ought to subsidize C and D B. Wages ought to be independent of international trade C. It is impossible to attain national self sufficiency D. The varying productivity of the various industries leads tot he inequalities in wages of workers in these industries E. A policy that draws labour from the fields of catater natural productiveness to fields of lower natural productiveness tends to redirect purchasing power. Ans : E

2. No employer, without government aid will yet be able to afford to hire labour to exploit the opportunities represented by C and D because A. The population has increased B. Productivity of labour is not at the maximum C. Productivity would drop correspondingly with the wages of labour D. We cannot produce everything we consume E. Enterprises would have to pay wages equivalent to those obtained by workers in A and B while producing under catater disadvantages. Ans : E 3. When C and D are established, workers in these industries A. Receives wages equal to those workers in A and B B. Receives higher wages than do the workers in A and B C. Are not affected so adversely by the levying of duties as are workers in A and B D. Must be paid by government funds collected from the duties on imports. E. Receive lower wages than do the workers in A and B. Ans : A 4. We cannot exploit C and D unless A. The producers in E and D are compensated for the disadvantages under which they operate. B. We export large quantities of commodities produced by A and B C. The prices of commodities produced by A and B are raised D. The productivity of labour in all industries is increased E. We allow duties to be paid to the producers in C and D rather than to the government. Ans : A

19. MARK HUGHES is a master of the fine art of survival. His Los Angeles-based Herbalife International Inc. is a pyramid outfit that peddles weight-loss and nutrition concoctions of dubious value. Bad publicity and regulatory crackdowns hurt his U.S. business in the late 1980s. But Hughes, 41, continues to enjoy a luxurious lifestyle in a $20 million Beverly Hills mansion. He has been sharing the pad and a yacht with his third wife, a former Miss Petite U.S.A. He can finance this lavish lifestyle just on his salary and bonus, which last year came to $7.3 million. He survived his troubles in the U.S. by moving overseas, where regulators are less zealous and consumers even more naive, at least initially. Today 77% of Herbalife retail sales derive from overseas. Its new prowling grounds: Asia and Russia. Last year Herbalife's net earnings doubled, to $45 million, on net sales of $632 million. Based on Herbalife's Nasdaq-traded stock, the company has a market capitalization of $790 million, making Hughes 58% worth $454 million.

There's a worm, though, in Hughes apple. Foreigners aren't stupid. In the end they know when they've been had. In France, for instance, retail sales rose to $97 million by 1993 and then plunged to $12 million last year. In Germany sales hit $159 million in 1994 and have since dropped to $54 million. Perhaps aware that the world may not provide an infinite supply of suckers, Hughes wanted to unload some of his shares. But in March, after Herbalife's stock collapsed, he put off a plan to dump about a third of his holdings on the public. Contributing to Hughes' woes, Herbalife's chief counsel and legal attack dog, David Addis, quit in January. Before packing up, he reportedly bellowed at Hughes, "I can't protect you anymore." Addis, who says he wants to spend more time with his family, chuckles and claims attorney-client privilege. Trouble on the home front, too. On a recent conference call with distributors, Hughes revealed he's divorcing his wife, Suzan, whose beaming and perky image adorns much of Herbalife's literature. Meanwhile, in a lawsuit that's been quietly moving through Arizona's Superior Court, former Herbalife distributor Daniel Fallow of Sandpoint, Idaho charges that Herbalife arbitrarily withholds payment to distributors and marks up its products over seven times the cost of manufacturing. Fallow also claims Hughes wanted to use the Russian mafia to gain entry to that nation's market. Fallow himself is no angel, but his lawsuit, which was posted on the Internet, brought out other complaints. Randy Cox of Lewiston, Idaho says Herbalife "destroyed my business" after he and his wife complained to the company that they were being cheated out of their money by higher-ups in the pyramid organization. Will Hughes survive again? Don't count on it this time. 1. Herbalife Inc is based in: A. Los Angeles B. Columbus C. New York D. Austin Ans : A 2. Daniel Fallow: A. Was a former attorney for Hughes B. Was a former distributor of Herbalife C. Co-founded Herbalife D. Ran Herbalife's German unit Ans : B 3. Which of the following countries is mentioned where Hughes operated Herbalife?

A. India B. China C. Germany D. Ukraine Ans : C 4. The complaint of Randy Cox of Lewiston, Idaho, against Herbalife was: A. The company did not pay them their dues B. The products supplied by Hughes were inferior C. Their higher-ups in the pyramid cheated them D. Hughes had connections with the Russian mafia Ans : C 5. Which of the following countries is NOT mentioned in the passage? A. Russia B. USA C. France D. Italy Ans : D 6. In the year in which Hughes' salary and bonuses came to US$ 7.3 million, what was the retail sales for Herbalife in France? A. $12 million B. $159 million C. $54 million D. $97 million Ans :A 7. At the time when this article was written, if Herbalife had had a market capitalisation of $ 1 billion, what would have been Hughes' share? A. $420 million B. $580 million C. $125 million D. $500 million Ans : B

WRITING SAMPLE:
The LSAT writing sample questions, which it is speculated is never looked at by the law schools, is still a required part of the test. The writing sample question poses a

situation where there are two equally weighted alternatives, and the goal is to make an argument about why one alternative should be chosen over the other. A good way to approach this section is to weigh the two options and then make a case for one over the other. (For example: While A provides such and such opportunity, B has the added bonus of such and such etc, therefore B is the better option). There is no right answer to this question: the options are presented to be equal. 1. Alice Anderson is a senior at John Paul Jones University. She has been offered two positions as a result of her outstanding record in her major, Television and Radio Broadcasting. As her counselor, you are to write an argument favoring one of the two offers. Two considerations guide your decision: o Alice has a large student loan debt that she has to begin to repay immediately upon graduation. o Alice has as her career goal a position as a network news anchorperson. LSAT writing sample passage is as follows: WAND is the only television station serving a large area located some 250 miles north of the capital of the state. The station has offered Alice a job as a reporter whose principal assignments would be to cover the activities of local governments, politics, and business. In addition to her assigned stories, Alice would have the opportunity to independently prepare stories for possible broadcast. Because the station is small, has a very stable staff, and has limited growth prospects, Alice's chances for advancement are not good. WAND's owner is a former network executive who purchased the station in order to get away from the pressures of broadcasting in major markets. Alice would get only a modest salary at WAND, and she would have to supplement her income with outside work. KBSC is one of three television stations located in the state capital. The station has offered Alice a job as a production assistant in the news department. She would primarily do background research and check facts and sources for the producers and reporters. Production assistants who work hard are promoted to positions as special assignment reporters in about two years. There are many special assignment reported competing for assignments, most of which involve covering minor events such as political dinners, award ceremonies, and concerts and writing human-interest stories. Most special assignment reporters spend at least five years covering minor events before moving into a position as a general report-anchorperson. KBSC would pay Alice a salary in excess of the amount she would need to live comfortably in the city. 2. Newhall City's television is choosing between two shows on steroid drug use. Write an argument supporting one show over the other based on the following considerations:

The station wants to attract younger viewers to its informational programs. The station wants its educational programs to be recognized for their in-depth analysis.

LSAT writing sample passage is as follows: "A Question of Health" is an investigation of teenage steroid use in Newhall City's four high schools narrated by Martene Seligman, prizewinning health columnist for the local newspaper. Seligman interviews a number of local students as well as school administrators, coaches, and faculty about the effects of steroid use on the students' lives. She also visits local hospitals and counseling centers to interview doctors and psychologists, who outline the serious physical and psychological effects of using steroids. Included in the show are detailed descriptions of treatment options available in Newhall City and their costs, as well as advice for parents who suspect their children of using steroids. "David's Game" follows the story of 16-year-old David Worsley, a high school track star from another city who was removed from the team and lost a college scholarship offer after using and selling steroids. A physician who worked with David is interviewed, but the show focuses primarily on David, his family, and his teammates, all of whom describe how David's increasing dependence on steroids gradually distorted both his appearance and his personality. Family and friends discuss their helplessness as they watched him become more aggressive and violent. David explains the pressures he felt to excel in sports, to the exclusion of all other interests, and he suggest ways in which the larger culture intensified this pressure. David ends the show by directing a plea to other teenagers to avoid steroids. 3. As the manager of the Pelican Beach, you have to support one of the two suggestions for improving the congested, overused and damaged state of Pelican Beach that has resulted from recent increases in beach use. One suggestion is to reduce the number of open beach areas and force users to book their area of the beach in advance. The other suggestion is to increase the number beaches open to the public and charge admission for the use of the beach. You must make your decision based on two factors:

Encouraging more people to the beach area. Preserving the natural state of the beach for public enjoyment.

By restricting beach usage to a smaller area, damage would be confined to a smaller area of the beach, and it would be easier to enforce existing rules. The reservation system would eliminate overcrowding in the beach areas. But, by confining users to certain areas, beach users do not get to use the full Pelican Beach area. By increasing the number of beach areas open to the public, more people would be able to surf, suntan and use the beach. Areas of damage would be less concentrated and easier to repair. The admissions charge would fund much-needed repairs. However, the fee would probably reduce the number of people going to Pelican Beach and the larger area of beach would make it more difficult to enforce rules.

S-ar putea să vă placă și